4410: Exam 1- Perfusion and Infection

¡Supera tus tareas y exámenes ahora con Quizwiz!

________________ is the amount the ventricle stretches at the end of diastole. A. Preload B. Afterload C. Stroke Volume D. Contractility

The answer is A. Preload is the amount the ventricle stretches at the end of diastole (hence it's the amount the ventricles stretches once it's filled with blood and right before the contraction of the ventricle ....so it's the end-diastolic volume).

They physician orders a Dobutamine IV drip on a patient in cardiogenic shock. After starting the IV drip, the nurse would make it priority to monitor for? A. Rebound hypertension B. Ringing in the ears C. Worsening hypotension D. Severe headache

The answer is C. Dobutamine increases contractility and cardiac output, BUT causes vasodilation due to the way it acts on receptors and this may make hypotension WORSE. The patient may be started on norepinephrine or dopamine if worsening of hypotension occurs.

During your morning assessment of a patient with heart failure, the patient complains of sudden vision changes that include seeing yellowish-green halos around the lights. Which of the following medications do you suspect is causing this issue? A. Lisinopril B. Losartan C. Lasix D. Digoxin

The answer is D. Yellowish-green halos/vision changes are classic signs of Digoxin toxicity.

True or False: Most patients with hypertension are asymptomatic.

True (HTN is called "the silent killer")

What are the 5 elements to the preventing ventilator-associated pneumonia (VAP) bundle?

1. HOB elevated to 45 degrees or more 2. Oral care with chlorhexidine 3. Stress ulcer prophylaxis meds (PPIs etc) 4. Daily sedation vacation and assessments 5. DVT prophylaxis (SCDs or meds)

The nurse is assessing a client diagnosed with cellulitis of the upper left arm. Which manifestation should the nurse anticipate finding with this​ client? (Select all that​ apply.) A. Swollen lymph glands B. Pustules with surrounding erythema C. ​Deep, firm, painful nodule D. Fever and chills E. Erythema

A, D, E

A client is admitted with cellulitis. Which manifestation should the nurse​ monitor? (Select all that​ apply.) A. Fever B. Chills C. Itching D. Headache E. Malaise

All but C

Which foods would the nurse encourage patients at risk for coronary artery disease (CAD) to include in their diets? (Select all that apply.) A. Tofu B. Walnuts C. Tuna fish D. Whole milk E. Orange juice

Answer: A, B, C Rationale: Tuna fish, tofu, and walnuts are all rich in omega-3 fatty acids, which have been shown to reduce the risks associated with CAD when consumed regularly.

A woman with severe preeclampsia is receiving a magnesium sulfate infusion. The nurse becomes concerned after assessment when the woman exhibits: A. a sleepy, sedated affect. B. a respiratory rate of 10 breaths/min. C. deep tendon reflexes of 2+. D. absent ankle clonus.

Answer: B Because magnesium sulfate is a central nervous system (CNS) depressant, the client will most likely become sedated when the infusion is initiated. A respiratory rate of 10 breaths/min indicates that the client is experiencing respiratory depression (bradypnea) from magnesium toxicity. Deep tendon reflexes of 2+ are a normal finding. Absent ankle clonus is a normal finding.

Weight loss of 22 lb (10 kg ) may decrease SBP by approx. ____ to ____ mm Hg A. 1 to 2 B. 5 to 20 C. 20 to 50

Answer: B. 5-20 mm Hg

A patient taking Simvastatin (Zocor) is reporting muscle pain. You are evaluating the patient's lab work and note that which of the following findings could cause muscle pain? A. Elevated potassium level B. Elevated CPK (creatine kinase level) C. Decreased potassium level D. Decreased CPK (creatine kinase level)

Answer: B. Simvastatin (Zocor) can cause increased CPK levels which will lead to a patient experiencing muscle pain. Therefore, CPK levels must be monitored while a patient is taking this medication.

Which coronary artery provides blood to the left atrium and left ventricle: A. Right marginal artery B. Posterior descending artery C. Left circumflex artery D. Right coronary artery

Answer: C. The LCA provides blood to the left atrium and left ventricle.

Select all the correct statements about the pharmacodynamics of Beta-blockers for the treatment of heart failure: A. These drugs produce a negative inotropic effect on the heart by increasing myocardial contraction. B. A side effect of these drugs include bradycardia. C. These drugs are most commonly prescribed for patients with heart failure who have COPD. D. Beta-blockers are prescribed with ACE or ARBs to treat heart failure. The answers are B and D.

B, D

For which problem is percutaneous coronary intervention (PCI) most clearly indicated? A. Chronic stable angina B. Left-sided heart failure C. Coronary artery disease D. Acute myocardial infarction

Correct Answer: Acute myocardial infarction Rationale: PCI is indicated to restore coronary perfusion in cases of myocardial infarction. Chronic stable angina and coronary artery disease are normally treated with more conservative measures initially. PCI is not relevant to the pathophysiology of heart failure.

When planning emergent care for a patient with a suspected myocardial infarction (MI), what should the nurse anticipate administering? A. Oxygen, nitroglycerin, aspirin, and morphine B. Aspirin, nitroprusside, dopamine, and oxygen C. Oxygen, furosemide (Lasix), nitroglycerin, and meperidine D. Nitroglycerin, lorazepam (Ativan), oxygen, and warfarin (Coumadin)

Correct Answer: Oxygen, nitroglycerin, aspirin, and morphine Rationale: The American Heart Association's guidelines for emergency care of the patient with chest pain include the administration of oxygen, nitroglycerin, aspirin, and morphine. These interventions serve to relieve chest pain, improve oxygenation, decrease myocardial workload, and prevent further platelet aggregation. The other medications may be used later in the patient's treatment.

Treatment modalities for the management of cardiogenic shock include (select all that apply) a. dobutamine to increase myocardial contractility. b. vasopressors to increase systemic vascular resistance. c. circulatory assist devices such as an intraaortic balloon pump. d. corticosteroids to stabilize the cell wall in the infarcted myocardium. e. Trendelenburg positioning to facilitate venous return and increase preload.

Correct answer: a, c

An expected finding in the assessment of an 81-year-old patient is a. a narrowed pulse pressure. b. diminished carotid artery pulses. c. difficulty isolating the apical pulse. d. an increased heart rate in response to stress.

Correct answer: c Rationale: Myocardial hypertrophy and the downward displacement of the heart in an older adult may cause difficulty in isolating the apical pulse.

A patient is admitted to the ICU with a diagnosis of NSTEMI. Which drugs(s) would the nurse expect the patient to receive? (select all that apply) a. Oral statin therapy b. Antiplatelet therapy c Thrombolytic therapy d. Prophylactic antibiotics e. Intravenous nitroglycerin

Correct answers: a, b, e Rationale: When a patient presents with suspected ACS, antiplatelet therapy, IV NTG, and atorvastatin are drug treatments of choice. For patients with UA and NSTEMI, heparin (UH or LMWH) is recommended to prevent microemboli from forming and causing further chest pain. DAPT (e.g., aspirin and clopidogrel or ticagrelor [Brilinta]) also is recommended for NSTEMI patients (with or without a stent). Thrombolytic therapy and antibiotics are not indicated for NSTEMI.

.The nurse is caring for a woman who is at 24 weeks of gestation with suspected severe preeclampsia. Which signs and symptoms would the nurse expect to observe? (Select all that apply.) A. Decreased urinary output and irritability B. Transient headache and +1 proteinuria C. Ankle clonus and epigastric pain D. Platelet count of less than 100,000/mm3 and visual problems E. Seizure activity and hypotension

Correct: A, C, D Rationale: Decreased urinary output and irritability are signs of severe eclampsia. Ankle clonus and epigastric pain are signs of severe eclampsia. Platelet count of less than 100,000/mm3 and visual problems are signs of severe preeclampsia. A transient headache and +1 proteinuria are signs of preeclampsia and should be monitored. Seizure activity and hyperreflexia are signs of severe eclampsia.

A client is admitted to the emergency department with chest pain and shortness of breath. An electrocardiogram indicates that the client is experiencing a myocardial infarction. An emergency cardiac catheterization is scheduled. What information should the nurse include in the preprocedure teaching? 1 Mild sedation is maintained during the procedure. 2 The procedure will take approximately 15 minutes to complete. 3 Ambulation is encouraged shortly after the procedure. 4 It will take approximately 24 hours to determine whether blockage is present.

1 A mild sedative is used because the client must be alert enough during the procedure to follow directions. A cardiac catheterization takes approximately 2 hours, not 15 minutes. The client remains on bed rest with the legs extended for 4 to 6 hours after the femoral method of entry. Blockages can be visualized during the procedure.

An infant with bronchiolitis caused by respiratory syncytial virus (RSV) is admitted to the pediatric unit. What does the nurse expect the prescribed treatment to include? 1 Humidified cool air and adequate hydration 2 Postural drainage and oxygen by hood 3 Bronchodilators and cough suppressants 4 Corticosteroids and broad-spectrum antibiotics

1 Humidified cool air and hydration are essential to facilitating improvement in the child's physical status. Postural drainage is not effective with this disorder; oxygen is used only if the infant has severe dyspnea and hypoxia. Bronchodilators are not used, because the bronchial tree is not in spasm; cough suppressants are ineffective. Corticosteroids are ineffective; antibiotics are also ineffective, because the causative agent is viral.

A client who is receiving multiple medications for a myocardial infarction complains of severe nausea, and the client's heartbeat is irregular and slow. The nurse determines that these signs and symptoms are toxic effects of what drug? 1Digoxin 2Captopril 3Furosemide 4Morphine sulfate

1 Signs of digoxin toxicity include cardiac dysrhythmias, anorexia, nausea, vomiting, and visual disturbances. Although nausea and heart block may occur with captopril, these symptoms rarely are seen; drowsiness and central nervous system disturbances are more common. Toxic effects of morphine are slow, deep respirations, stupor, and constricted pupils; nausea is a side effect, not a toxic effect. Toxic effects of furosemide are renal failure, blood dyscrasias, and loss of hearing. Term

Nitroglycerin sublingual tablets are prescribed for a client with the diagnosis of angina. The client asks the nurse how long it should take for the chest pain to subside after nitroglycerin is taken. What should the nurse tell the client? 1. 1 to 3 minutes 2. 4 to 5 seconds 3. 30 to 45 seconds 4. 20 to 45 minutes

1 The onset of action of sublingual nitroglycerin tablets is rapid (1 to 3 minutes); duration of action is 30 to 60 minutes. If nitroglycerin is administered intravenously, the onset of action is immediate, and the duration is 3 to 5 minutes. It takes longer than 30 to 45 seconds for sublingual nitroglycerin tablets to have a therapeutic effect. Sustained-release nitroglycerin tablets start to act in 20 to 45 minutes, and the duration of action is 3 to 8 hours.

A nurse is caring for a client with a diagnosis of right ventricular heart failure. The nurse expects what assessment findings associated with right-sided heart failure? Select all that apply. 1 Dependent edema 2 Swollen hands and fingers 3 Collapsed neck veins 4 Right upper quadrant discomfort 5 Oliguria

1,2,4 With right-sided heart failure, signs of systemic congestion occur as the right ventricle fails; key features include dependent edema and swollen hands and fingers. Upper right quadrant discomfort is expected with right ventricular failure because venous congestion in the systemic circulation results in hepatomegaly. Jugular venous collapse and oliguria are key features of left-sided heart failure. Left-sided heart failure is associated with decreased cardiac output.

After reviewing the urinalysis reports of a client with a renal disorder, the nurse concludes that the client may have a urinary tract infection. Which urinary laboratory findings enabled the nurse to make this conclusion? Select all that apply. 1 pH: 8.5 2 Specific gravity: 1.010 3 Red blood cells: 3/hpf 4 Osmolality: 1500 mOsm/kg (1500 mmol/kg) 5 White blood cells: 6/hpf

1,5 The client may have a urinary tract infection, as the urinalysis reports show the presence of pH as 8.5 and white blood cells as 6/hpf in the urine. A pH above 8.0 indicates a urinary tract infection; client's is 8.5. The normal level of white blood cells (WBC) in urine should be less than 5/hpf; therefore, the WBC level of 6/hpf indicates urinary tract infection. The specific gravity of 1.010 indicates a normal finding. The normal level of red blood cells (RBC) is less than 4/hpf; therefore, the RBC levels of 3/hpf indicates normal finding. Osmolality of 1500 mOsm/kg (1500 mmol/kg) indicates tubular dysfunction.

The nurse is caring for a client who had a massive myocardial infarction and developed cardiogenic shock. Which clinical manifestations support these diagnoses? Select all that apply. 1 Rapid pulse 2 Deep respirations 3 Warm, flushed skin 4 Increased blood pressure 5 Decreased urinary output

1,5 The heart rate increases (tachycardia) in an attempt to meet the body's oxygen demands and circulate blood to vital organs; the pulse is weak and thready because of peripheral vasoconstriction. The urinary output decreases because increased catecholamines and activation of the renin-angiotensin-aldosterone system increase fluid reabsorption in the kidneys. The respirations are rapid and shallow, not deep. The skin is cold and clammy because of vasoconstriction caused by the shunting of blood to vital organs. The blood pressure is decreased, not increased, because of continued hypoperfusion and multiorgan failure.

The nurse is preparing discharge instructions for a client who was prescribed enalapril for treatment of hypertension. Which instruction is appropriate for the nurse to include in the client's teaching? 1. Do not change to a standing position suddenly. 2. Lightheadedness is a common adverse effect that need not be reported. 3. The medication may cause a sore throat for the first few days. 4. Schedule blood tests weekly for the first 2 months.

1. Enalapril is classified as an angiotensin-converting enzyme (ACE) inhibitor. It is used to treat hypertension and congestive heart failure. It can also be used to treat a disorder of the ventricles. Angiotensin is a chemical that causes the arteries to become narrow. ACE inhibitors help the body produce less angiotensin, which helps the blood vessels relax and open up, which, in turn, lowers blood pressure. Clients should be advised to change positions slowly to minimize orthostatic hypotension. A healthcare provider should be notified immediately if the client is experiencing lightheadedness or feeling like he or she is about to faint, as this is a serious side effect. This medication does not cause a sore throat the first few days of treatment. Presently, there are no guidelines that suggest blood tests are required weekly for the first 2 months.

A patient with type 2 diabetes is reporting a second urinary tract infections(UTI)within the past month. Which medication should the nurse expect to be ordered for the recurrent infection? 1. Ciprofloxacin 2. Fosfomycin 3. Nitrofurantoin 4. Trimethoprim-sulfamethoxazole

1. This UTI is a complicated UTI because the patient has type 2 diabetes, and the UTI is recurrent. Ciprofloxacin would be used for a complicated UTI. Fosfomycin, nitrofurantoin , and trimethoprim-sulfamethoxazole should be used for uncomplicated UTIs.

A nurse is caring for a client who has had multiple myocardial infarctions and has now developed cardiogenic shock. Which clinical manifestation supports this diagnosis? 1. Cold, clammy skin 2. Slow, bounding pulse 3. Increased blood pressure 4. Hyperactive bowel sounds

1. cold clammy skin The action of the sympathetic nervous system causes vasoconstriction, and as cellular and peripheral hypoperfusion progresses, the skin becomes cold, clammy, cyanotic, or mottled. The heart rate increases in an attempt to meet the body's oxygen demands and circulate blood to vital organs; it has a low volume (weak, thready) because of peripheral vasoconstriction. The blood pressure decreases because of continued hypoperfusion and multiorgan failure. Bowel sounds are hypoactive or absent, not hyperactive

A client who recently had a myocardial infarction is admitted to the cardiac care unit. How can the nurse best determine the effectiveness of the client's ventricular contractions? 1 Observing anxiety levels 2 Monitoring urinary output hourly 3 Evaluating cardiac enzyme results 4 Assessing breath sounds frequently

2 A decreased urinary output reflects a decreased cardiac output; immediate action is indicated if urinary output decreases. Although anxiety may occur, the priority is to monitor urinary output, which reflects cardiac effectiveness. Cardiac enzyme results do not reflect effectiveness of cardiac contractions; they reflect tissue damage. Although the presence of crackles (rales) will indicate pulmonary edema, it will not determine the effectiveness of ventricular contractions.

The nurse is performing an assessment for a patient undergoing radiation treatment for breast cancer. What position should the nurse place the patient to best auscultate for signs of acute pericarditis? 1 Supine without a pillow 2 Sitting and leaning forward 3 Left lateral side-lying position 4 Head of bed at a 45-degree angle

2 A pericardial friction rub indicates pericarditis. To auscultate a pericardial friction rub, the patient should be sitting and leaning forward. The nurse will hear the pericardial friction rub at the end of expiration.

A client who is suspected of having had a silent myocardial infarction has an electrocardiogram (ECG) prescribed by the primary healthcare provider. While the nurse prepares the client for this procedure, the client asks, "Why was this test prescribed?" Which is the best reply by the nurse? 1 "This test will detect your heart sounds." 2 "This test will reflect any heart damage." 3 "This procedure helps us change your heart's rhythm." 4 "The ECG will tell us how much stress your heart can tolerate."

2 Changes in an ECG will reflect the area of the heart that is damaged because of hypoxia. A stethoscope is used to detect heart sounds. Medical interventions, such as cardioversion or cardiac medications, not an ECG, can alter heart rhythm. An ECG will reflect heart rhythm, not change it. Identifying how much stress a heart can tolerate is accomplished through a stress test; this uses an ECG in conjunction with physical exercise.

A client is admitted to the hospital with a diagnosis of heart failure and acute pulmonary edema. The healthcare provider prescribes furosemide 40 mg intravenous (IV) stat to be repeated in 1 hour. What nursing action will best evaluate the effectiveness of the furosemide in managing the client's condition? 1 Perform daily weights 2 Auscultate breath sounds 3 Monitor intake and output 4 Assess for dependent edema

2 Maintaining adequate gas exchange and minimizing hypoxia with pulmonary edema are critical; therefore, assessing the effectiveness of furosemide therapy as it relates to the respiratory system is most important. Furosemide inhibits the reabsorption of sodium and chloride from the loop of Henle and distal renal tubule, causing diuresis; as diuresis occurs fluid moves out of the vascular compartment, thereby reducing pulmonary edema and the bilateral crackles. Although a liter of fluid weighs approximately 2.2 pounds (1 kilogram) and weight loss will reflect the amount of fluid lost, it will take time before a change in weight can be measured. Although identifying a greater output versus intake indicates the effectiveness of furosemide, it is the client's pulmonary status that is most important with acute pulmonary edema. Although the lessening of a client's dependent edema reflects effectiveness of furosemide therapy, it is the client's improving pulmonary status that is most important.

The patient informs the nurse that he does not understand how there can be a blockage in the left anterior descending artery (LAD), but there is damage to the right ventricle. What is the best response by the nurse? 1 "The one vessel curves around from the left side to the right ventricle." 2 "The LAD supplies blood to the left side of the heart and part of the right ventricle." 3 "The right ventricle is supplied during systole primarily by the right coronary artery." 4 "It is actually on your right side of the heart, but we call it the left anterior descending vessel."

2 The best response is explaining that the lower portion of the right ventricle receives blood flow from the left anterior descending artery as well as the right coronary artery during diastole.

A client is considered to be in septic shock when what changes are assessed in the client's labwork? 1 Blood glucose is 70-100 mg/dL 2 An increased serum lactate level 3 An increased neutrophil level 4 A white blood count of 5000 cells/µL

2 The hallmark of sepsis is an increasing serum lactate level, a normal or low total WBC count > 12,000 cells/µL or < 4,000 cells/µL and a decreasing segmented neutrophil level with a rising band neutrophil level. Blood glucose levels with sepsis are between 110 and >150 mg/dL. Blood glucose levels of 70-100 mg/dL are considered normal. Term

Trimethoprim/sulfamethoxazole is prescribed for a child with a urinary tract infection. Which statement by the parent about the drug indicates that the nurse's instructions about administration have been understood? 1 "Mealtime is a good time to give the medication." 2 "I'll make sure to give each pill with 6 to 8 oz (180 to 240 mL) of fluid." 3 "It must be taken with orange juice to ensure acidity of urine." 4 "The drug has to be taken every 4 hours to maintain a blood level."

2 This is a sulfa drug; water must be encouraged to prevent urine crystallization in the kidneys. This drug does not have to be given with meals; it is administered every 12 hours. Orange juice causes an alkaline urine; water is the best fluid to be administered with this drug. This drug maintains the blood level for 8 to 12 hours; it is an intermediate-acting drug.

An infant is admitted to the pediatric unit with bronchiolitis caused by respiratory syncytial virus (RSV). What interventions are appropriate nursing care for the infant? Select all that apply. 1 Limiting fluid intake 2 Instilling saline nose drops 3 Maintaining contact precautions 4 Suctioning mucus with a bulb syringe 5 Administering warm humidified oxygen

2,3,4 Saline nose drops help clear the nasal passage, which improves breathing and aids the intake of fluids. RSV is contagious; infants with RSV should be isolated from other children, and the number of people visiting or caring for the infant should be limited. Infants with RSV produce copious amounts of mucus, which hinders breathing and feeding; suctioning before meals and at naptime and bedtime provides relief. Fluid intake should be increased; adequate hydration is essential to counter fluid loss. These infants have difficulty nursing and often vomit their feedings. If measures such as suctioning before feeding and instilling saline nose drops are ineffective, intravenous fluid replacement is instituted. The humidified oxygen should be cool. It relieves the dyspnea and hypoxia that is prevalent in infants with RSV.

The nurse is planning to teach a client with heart failure about the signs and symptoms of cardiac decompensation. What clinical manifestations should the nurse include? Select all that apply. 1 Weight loss 2 Extreme fatigue 3 Coughing at night 4 Excessive urination 5 Difficulty breathing

2,3,5. Fatigue is caused by a lack of adequate oxygenation of body cells caused by a decreased cardiac output. As the cardiac output decreases, pulmonary congestion increases, resulting in pulmonary edema; coughing, especially when lying down, and blood-tinged sputum occur. Auscultation reveals crackles and rhonchi. Dyspnea is associated with pulmonary edema that occurs as cardiac output decreases and pulmonary congestion increases. Weight gain, not loss, occurs as fluid is retained by the kidneys. Fluid retention, not diuresis, occurs because of decreased circulation to the kidneys, resulting from decreased cardiac output.

Which intervention is most likely to decrease mortality in the septic client? 1. Oxygen 2. Antibiotics 3. Vasopressors 4. Intravenous fluids

2. Of the interventions listed, administering antibiotics is the only intervention that fights the source of the problem. Intravenous fluids, oxygen, and vasopressors are necessary, but are designed to sustain the body until the antibiotic can kill the pathogen.

The nurse is caring for a group of clients who have sustained myocardial infarction (MI). The nurse observes the client with which type of MI most carefully for the development of left ventricular heart failure? 1. Inferior wall 2. Anterior wall 3. Lateral wall 4. Posterior wall

2. Owing to the large size of the anterior wall, the amount of tissue infarction may be large enough to decrease the force of contraction, leading to heart failure. with the inferior wall, the client is more likely to develop right ventricular MI. regarding clients with obstruction of the circumflex artery may experience a lateral wall or posterior wall MI and sinus dysrhythmias.

A 6-month-old infant is brought to the emergency department in severe respiratory distress. A diagnosis of respiratory syncytial virus (RSV) infection is made, and the infant is admitted to the pediatric unit. What should be included in the nursing plan of care? 1. Place in a warm, dry environment. 2. Maintain standard and contact precautions. 3. Administer prescribed antibiotic immediately. 4. Allow parents and siblings to room in with the infant.

2. RSV is highly contagious. The infant should be isolated or placed with other infants with RSV. Standard and contact precautions are instituted to limit the spread of pathogens to others. The infant should receive cool, humidified oxygen by nasal cannula or mask or in a croup tent. Because RSV is extremely contagious, the number of visitors should be limited. Uninfected children should not be allowed near the infant, and as few personnel as possible should care for the infant. Antibiotics are not effective against RSV, and their use is contraindicated.

A client with hypertensive heart disease, who had an acute episode of heart failure, is to be discharged on a regimen of metoprolol and digoxin. What outcome does the nurse anticipate when metoprolol is administered with digoxin? 1. Headaches 2. Bradycardia 3. Hypertension 4. Junctional tachycardia

2. bradycardia Metoprolol and digoxin both exert a negative chronotropic effect, resulting in a decreased heart rate. Metoprolol reduces, not produces, headaches. These drugs may cause hypotension, not hypertension. These drugs may depress nodal conduction; therefore, junctional tachycardia would be less likely to occur.

A client with a history of angina is scheduled for a cardiac catheterization. Catheter entry will be through the femoral artery. What should the nurse tell the client to expect? 1 Remain fully alert during the procedure 2 Ambulate shortly after the procedure 3 Experience a feeling of warmth during the procedure 4 Be placed in a semi-Fowler position for 12 hours after the procedure

3 A warm flushing sensation that lasts approximately 30 seconds will occur when the contrast medium is injected. Medication is given for mild sedation; clients are drowsy but awake enough to follow instructions. The supine position will be maintained for 4 to 6 hours after the procedure; walking may dislodge clots at the catheter insertion site, resulting in bleeding. The semi-Fowler position flexes the legs, which may result in bleeding at the femoral insertion site, and should be avoided. Term

A client with a long history of cardiovascular problems, including angina and hypertension, is scheduled to have a cardiac catheterization. During preprocedure teaching, what does the nurse explains to the client is the major purpose for catheterization? 1 To obtain the pressures in the heart chambers 2 To determine the existence of congenital heart disease 3 To visualize the disease process in the coronary arteries 4 To measure the oxygen content of various heart chambers

3 Angina usually is caused by narrowing of the coronary arteries; the lumen of the arteries can be assessed by cardiac catheterization. Although pressures can be obtained, they are not the priority for this client; this assessment is appropriate for those with valvular disease. Determining the existence of congenital heart disease is appropriate for infants and young adults with cardiac birth defects. Measuring the oxygen content of various heart chambers is appropriate for infants and young children with suspected septal defects.

Which intervention would be most beneficial in preventing a catheter-associated urinary tract infection in a postoperative client? 1 Pouring warm water over the perineum 2 Ensuring the patency of the catheter 3 Removing the catheter within 24 hours 4 Cleaning the catheter insertion site

3 Clients who undergo surgery are at a greater risk of acquiring catheter-associated urinary tract infections. Infections can be prevented by removing the catheter within 24 hours, if the client does not need it. Therefore removing the catheter within 24 hours would be the best intervention. While pouring warm water over the perineum helps voiding in the postoperative client and also reduces the chances of infection, this action would not be as beneficial as the former intervention. The catheter should be maintained in its place to avoid leakage and infection. Cleaning the catheter insertion site will definitely reduce the risk of infection, but this action cannot prevent infections if the catheter is inserted for a long time.

A nurse discovers lower extremity pitting edema in a client with right ventricular heart failure. Which information should the nurse consider when planning care? 1Client has decreased plasma colloid osmotic pressure. 2Client has increased tissue colloid osmotic pressure. 3Client has increased plasma hydrostatic pressure. 4Client has decreased tissue hydrostatic pressure.

3 In right ventricular heart failure, blood backs up in the systemic capillary beds; the increase in plasma hydrostatic pressure shifts fluid from the intravascular compartment to the interstitial spaces, causing edema. Increase in tissue (interstitial) colloid osmotic pressure occurs with crushing injuries or if proteins pathologically shift from the intravascular compartment to the interstitial spaces, pulling fluid and causing edema. In right ventricular heart failure, increased fluid pressure in the intravascular compartment causes fluid to shift to the tissues; the tissue hydrostatic pressure does not decrease. Although a decrease in colloid osmotic (oncotic) pressure can cause edema, it results from lack of protein intake, not increased hydrostatic pressure associated with right ventricular heart failure. Term

A client who is hospitalized after a myocardial infarction asks the nurse why morphine was prescribed. What will the nurse include in the reply? 1 Decreases anxiety and promotes sleep 2 Helps prevent the development of atrial fibrillation 3 Relieves pain and reduces cardiac oxygen demand 4 Dilates coronary blood vessels to increase oxygen supply

3 Morphine is a specific central nervous system depressant used to relieve the pain associated with myocardial infarction; it also decreases apprehension and reduces cardiac oxygen demand by decreasing cardiac workload. Dilating coronary blood vessels is not the reason for the use of morphine. Decreasing anxiety and restlessness is not the primary reason for the use of morphine. Lidocaine is given intravenously to prevent fibrillation of the heart. Term

To prevent septic shock in the hospitalized client, what should the nurse do? 1 Maintain the client in a normothermic state. 2 Administer blood products to replace fluid losses. 3 Use aseptic technique during all invasive procedures. 4 Keep the critically ill client immobilized to reduce metabolic demands.

3 Septic shock occurs as a result of an uncontrolled infection, which may be prevented by using correct infection control practices. These include aseptic technique during all invasive procedures. Maintaining the client in a normothermic state, administering blood products, and keeping the critically ill client immobilized are not directly related to the prevention of septic shock.

The nurse provides discharge teaching to a client with a history of angina. The nurse instructs the client to call for emergency services immediately if the client's pain exhibits which characteristic? 1 Causes mild perspiration 2 Occurs after moderate exercise 3 Continues after rest and nitroglycerin 4 Precipitates discomfort in the arms and jaw

3 When neither rest nor nitroglycerin relieves the pain, the client may be experiencing an acute myocardial infarction. Angina may cause mild diaphoresis; acute myocardial infarction causes profuse diaphoresis, which should be reported. Chest pain after exercise is expected; activity increases cardiac output, which can cause angina. Anginal pain can, and often does, radiate. Term

A patient with a history of chronic hypertension is being evaluated in the emergency department for a blood pressure of 200/140 mm Hg. Which patient assessment question is the priority? 1. Is the patient pregnant? 2. Does the patient need to urinate? 3. Does the patient have a headache or confusion? 4. Is the patient taking antiseizure medications as prescribed?

3. The nurse's priority assessments include neurologic deficits, retinal damage, heart failure, pulmonary edema, and renal failure. The headache or confusion could be seen with hypertensive encephalopathy from increased cerebral capillary permeability leading to cerebral edema. In addition, headache or confusion could represent signs and symptoms of a hemorrhagic stroke. Pregnancy can lead to secondary hypertension. Needing to urinate and taking antiseizure medication do not support a hypertensive emergency.

A healthcare provider prescribes milrinone for a client with a diagnosis of congestive heart failure who was unresponsive to conventional drug therapy. What is most important for the nurse to do first? 1 Administer the loading dose over 10 minutes. 2 Monitor the ECG continuously for dysrhythmias during infusion. 3 Assess the heart rate and blood pressure continuously during infusion. 4 Have the prescription, dosage calculations, and pump settings checked by a second nurse.

4 Accidental overdose can cause death. Another nurse should verify accuracy of the prescription, dose, and pump settings to prevent harm to the client. Although administering the loading dose over 10 minutes is an appropriate intervention, it is not the first thing the nurse should do. Although monitoring for dysrhythmias is important because they are common with this medication and may be life threatening, it is not the first thing the nurse should do. Although taking the vital signs continuously during the infusion is important because the dose needs be slowed or discontinued if the blood pressure decreases excessively, it is not the first thing the nurse should do.

What physiologic alteration does the nurse expect when assessing a 6-month-old infant with bronchiolitis (respiratory syncytial virus [RSV])? 1 Decreased heart rate 2 Intercostal retractions 3 Increased breath sounds 4 Prolonged expiratory phase

4 Infectious and mechanical changes narrow the bronchial passages and make it difficult for air to leave the lungs. As a result of increased respiratory effort and decreased oxygen exchange, tachycardia may develop. Intercostal retractions are unlikely because of overinflation of the chest with air and shallow, rapid breathing. Breath sounds may be diminished because of swelling of the bronchiolar mucosa and filling of the lumina with mucus and exudate. Term

A 67-yr-old woman with hypertension is admitted to the emergency department with a blood pressure of 234/148 mm Hg and was started on nitroprusside (Nitropress). After one hour of treatment, the mean arterial blood pressure (MAP) is 55 mm Hg. Which nursing action is a priority? 1 Start an infusion of 0.9% normal saline at 100 mL/hr. 2 Maintain the current administration rate of the nitroprusside. 3 Request insertion of an arterial line for accurate blood pressure monitoring. 4 Stop the nitroprusside infusion and assess the patient for potential complications

4 Nitroprusside is a potent vasodilator medication. A blood pressure of 234/118 mm Hg would have a calculated MAP of 177 mm Hg. Subtracting 25% (or 44 mm Hg) = 133 mm Hg. The initial treatment goal is to decrease MAP by no more than 25% within minutes to 1 hour. For this patient, the goal MAP would be approximately 133 mm Hg. Minimal MAP required to perfuse organs is around 60 to 65 mm Hg. Lowering the blood pressure too rapidly may decrease cerebral, coronary, or renal perfusion and could precipitate a stroke, myocardial infarction, or renal failure. The priority is to stop the nitroprusside infusion and then use fluids only if necessary to support restoration of MAP.

When a client with heart failure is seen in the clinic with new onset ankle edema, the nurse would question the client about which lifestyle factor that may have contributed to the ankle swelling? Select all that apply. One, some, or all responses may be correct. A. Intake of salty foods B. Increased fluid consumption C. Dietary fat intake D. Medication compliance E. Family stresses F. Recent travel G. Alcohol intake H. Increased physical activity

A, B, D, F, G Rationale: Fluid retention in heart failure may be caused by increased salt intake, with associated water retention. This may also be caused by increased dietary fluids. Poor adherence to medication used to treat heart failure, such as angiotensin-converting-enzyme inhibitors and diuretics, may also cause fluid retention. Recent travel may cause fluid retention because of changes in environmental temperature, effects of airplane travel on fluid retention, or changes in dietary sodium intake. Increased alcohol intake can worsen the effects of heart failure, including edema. Increased or decreased dietary fat intake will not cause fluid retention. Stress is not a contributor to fluid retention. The client with heart failure is encouraged to stay as active as possible, without overdoing it. A sedentary, not active, lifestyle would contribute to fluid accumulation.

The nurse is performing a health history for a new client in the clinic. Which should the nurse identify as a risk factor for cellulitis in an​ adult? (Select all that​ apply.) A. Peripheral vascular disease B. Hypertension C. Obesity D. Diabetes mellitus E. Impetigo

A, C, D

Evidence-based approaches to preventing central line-associated bloodstream infections (CLABSI) include: SATA A. Use appropriate hand hygiene. B. Use soap and water for skin preparation and dressing changes. C. Use full-barrier precautions during central venous catheter insertion (sterile procedure). D. Always using the femoral vein for catheters in adult patients. E. Remove unnecessary catheters.

A, C, E Use chlorhexidine for skin preparation in central lines. AVOID using the femoral vin for adults, the subclavian vein is preferred. Reasons to leave catheters in include: for certain meds (vasopressors-NE, dobutamine, epi), chemotherapy agents, or massive fluid administration

At a clinic visit, the nurse provides dietary teaching for a patient recently hospitalized with an exacerbation of chronic heart failure. The nurse determines that teaching is successful if the patient makes which statement? A. "I will limit the amount of milk and cheese in my diet." B. "I can add salt when cooking foods but not at the table." C. "I will take an extra diuretic pill when I eat a lot of salt." D. "I can have unlimited amounts of foods labeled as reduced sodium."

A. Milk products should be limited to 2 cups per day for a 2500-mg sodium-restricted diet. Salt should not be added during food preparation or at the table. Diuretics should be taken as prescribed (usually daily) and not based on sodium intake. Foods labeled as reduced sodium contain at least 25% less sodium than regular.

When caring for a patient in acute septic shock, what should the nurse anticipate? A. Infusing large amounts of IV fluids B. Administering osmotic and/or loop diuretics C. Administering IV diphenhydramine (Benadryl) D. Assisting with insertion of a ventricular assist device (VAD)

A. Septic shock is characterized by a decreased circulating blood volume. Volume expansion with the administration of IV fluids is the cornerstone of therapy. Administering diuretics is inappropriate. VADs are useful for cardiogenic shock, not septic shock. Diphenhydramine may be used for anaphylactic shock but would not be helpful with septic shock.

An asymptomatic patient with acute decompensated heart failure (ADHF) suddenly becomes dyspneic. Before dangling the patient on the bedside, what should the nurse assess first? A. Urine output B. Heart rhythm C. Breath sounds Blood pressure

A. Correct Answer: Blood pressure Rationale: The nurse should evaluate the blood pressure before dangling the patient on the bedside because the blood pressure can decrease as blood pools in the periphery and preload decreases. If the patient's blood pressure is low or marginal, the nurse should put the patient in the semi-Fowler's position and use other measures to improve gas exchange.

What should the nurse recognize as an indication for the use of dopamine in the care of a patient with heart failure? A. Acute anxiety B. Hypotension and tachycardia C. Peripheral edema and weight gain Paroxysmal nocturnal dyspnea (PND)

A. Correct Answer: Hypotension and tachycardia Rationale: Dopamine is a β-adrenergic agonist whose inotropic action is used for treatment of severe heart failure accompanied by hemodynamic instability. Such a state may be indicated by tachycardia accompanied by hypotension. PND, anxiety, edema, and weight gain are common signs and symptoms of heart failure, but these do not necessarily warrant the use of dopamine.

The nurse is caring for a patient in cardiogenic shock after an acute myocardial infarction. Which assessment findings would be most concerning? A. Restlessness, heart rate of 124 beats/min, and hypoactive bowel sounds B. Agitation, respiratory rate of 32 breaths/min, and serum creatinine of 2.6 mg/dL C. Mean arterial pressure of 54 mm Hg; increased jaundice; and cold, clammy skin D. PaO2 of 38 mm Hg, serum lactate level of 46.5 mcg/dL, and puncture site bleeding

A. PaO2 of 38 mm Hg, serum lactate level of 46.5 mcg/dL, and puncture site bleeding Severe hypoxemia, lactic acidosis, and bleeding are manifestations of the irreversible state of shock. Recovery from this stage is not likely because of multiple organ system failure. Restlessness, tachycardia, and hypoactive bowel sounds are manifestations that occur during the compensatory stage of shock. Decreased mean arterial pressure, jaundice, cold and clammy skin, agitation, tachypnea, and increased serum creatinine are manifestations of the progressive stage of shock.

After coronary artery bypass graft surgery, a patient has postoperative bleeding that requires returning to surgery for repair. During surgery, the patient has a myocardial infarction (MI). After restoring the patient's body temperature to normal, which patient parameter is the most important for planning nursing care? A. Cardiac index (CI) of 5 L/min/m2 B. Central venous pressure of 8 mm Hg C. Mean arterial pressure (MAP) of 86 mm Hg D. Pulmonary artery pressure (PAP) of 28/14 mm Hg

A. Pulmonary artery pressure (PAP) of 28/14 mm Hg Pulmonary hypertension as indicated by an elevated PAP indicates impaired forward flow of blood because of left ventricular dysfunction or hypoxemia. Both can be a result of the MI. The CI, CVP, and MAP readings are normal.

A client is admitted to the cardiac care unit with a myocardial infarction. The cardiac monitor reveals several runs of ventricular tachycardia. The nurse anticipates that the client will be receiving a prescription for which drug? 1. Atropine 2. Epinephrine 3. Amiodarone 4. Sodium bicarbonate

Amiodarone (class III anti-arrhythmic drug/ Potassium channel blocker) suppresses ventricular activity; therefore, it is used for treatment of premature ventricular complexes (PVCs). It works directly on the heart tissue and slows the nerve impulses in the heart. Atropine blocks vagal stimulation; it increases the heart rate and is used for bradycardia, not PVCs. Epinephrine increases myocardial contractility and heart rate; therefore, it is contraindicated in the treatment of PVCs. Sodium bicarbonate increases the serum pH level; therefore, it combats metabolic acidosis.

A woman with severe preeclampsia has been receiving magnesium sulfate by intravenous (IV) infusion for 8 hours. The nurse assesses the woman and documents the following findings: temperature 37.1° C, pulse rate 96 beats/min, respiratory rate 24 breaths/min, blood pressure 155/112 mm Hg, 3+ deep tendon reflexes, and no ankle clonus. The nurse calls the physician, anticipating an order for: A. hydralazine. B. magnesium sulfate bolus. C. diazepam. D. calcium gluconate.

Answer: A Hydralazine is an antihypertensive commonly used to treat hypertension in severe preeclampsia. An additional bolus of magnesium sulfate may be ordered for increasing signs of central nervous system irritability related to severe preeclampsia (e.g., clonus) or if eclampsia develops. Diazepam sometimes is used to stop or shorten eclamptic seizures. Calcium gluconate is used as the antidote for magnesium sulfate toxicity. The client is not currently displaying any signs or symptoms of magnesium toxicity.

Which antilipemic medications should the nurse question for a patient who has cirrhosis of the liver? (Select all that apply.) A. Niacin B. Cholestyramine C. Ezetimibe (Zetia) D. Gemfibrozil (Lopid) E. Atorvastatin (Lipitor)

Answer: A, C, D, E Rationale: Ezetimibe (Zetia) should not be used by patients with liver impairment. Adverse effects of atorvastatin (Lipitor), a statin drug, include liver damage and myopathy. Liver enzymes must be monitored frequently, and the medication stopped if these enzymes increase. Niacin's side effects subside with time, although decreased liver function may occur with high doses. Cholestyramine is safe for long-term use/ safe for liver disease pts.

The patient has heart failure (HF) with an ejection fraction of less than 40%. What core measures should the nurse expect to include in the plan of care for this patient? (Select all that apply.) A. Left ventricular function is documented B. Controlling dysrhythmias will eliminate HF C. Prescription for digoxin (Lanoxin) at discharge D. Prescription for angiotensin-converting enzyme inhibitor at discharge E. Education materials about activity, medications, weight monitoring, and what to do if symptoms worsen

Answer: A, D, E Rationale: The Joint Commission has identified these 3 core measures for heart failure patients. Although controlling dysrhythmias will improve CO and workload, it will not eliminate HF. Prescribing digoxin for all HF patients is no longer done because there are newer effective drugs and digoxin toxicity occurs easily related to electrolyte levels and the therapeutic range must be maintained.

A woman with severe preeclampsia is being treated with an intravenous (IV) infusion of magnesium sulfate. This treatment is considered successful if: A. blood pressure is reduced to prepregnant baseline. B. seizures do not occur. C. deep tendon reflexes become hypotonic. D. diuresis reduces fluid retention.

Answer: B A temporary decrease in blood pressure can occur; however, this is not the purpose of administering this medication. Magnesium sulfate is a central nervous system (CNS) depressant given primarily to prevent seizures. Hypotonia is a sign of an excessive serum level of magnesium. It is critical that calcium gluconate be on hand to counteract the depressant effects of magnesium toxicity. Diuresis is not an expected outcome of magnesium sulfate administration.

A normal PAWP is _____ mmHg and if it's ____mmHg this indicates cardiogenic shock. A. 10-20; >30 B. 20-30; >50 C. 4-12 ; >18 D. 1-5; >10

Answer: C PAWP or PCWP is Pulmonary Artery Wedge Pressure. The terms "pulmonary arterial wedge pressure" (PAWP) and "left ventricular end-diastolic pressure" (LVEDP) are often used interchangeably to describe left-sided filling pressures/ left atrial pressure/ assess mitral valve function. PAWP can be measured with a Swan-Ganz catheter by inserting the balloon-tipped, multi-lumen catheter into a peripheral vein (e.g., jugular or femoral vein), then advancing the catheter into the right atrium, right ventricle, pulmonary artery, and then into a branch of the pulmonary artery

What is the priority assessment by the nurse caring for a patient receiving IV nesiritide (Natrecor) to treat heart failure? A. Urine output B. Lung sounds C. Blood pressure D. Respiratory rate

Answer: C Rationale: Although all identified assessments are appropriate for a patient receiving IV nesiritide, the priority assessment would be monitoring for hypotension, the main adverse effect of nesiritide. Nesiritide is a vasodilator

A woman at 39 weeks of gestation with a history of preeclampsia is admitted to the labor and birth unit. She suddenly experiences increased contraction frequency of every 1 to 2 minutes; dark red vaginal bleeding; and a tense, painful abdomen. The nurse suspects the onset of: A. eclamptic seizure. B. rupture of the uterus. C. placenta previa. D. placental abruption.

Answer: D Eclamptic seizures are evidenced by the presence of generalized tonic-clonic convulsions. Uterine rupture presents as hypotonic uterine activity, signs of hypovolemia, and in many cases the absence of pain. Placenta previa presents with bright red, painless vaginal bleeding. Uterine tenderness in the presence of increasing tone may be the earliest finding of premature separation of the placenta (abruptio placentae or placental abruption). Women with hypertension are at increased risk for an abruption.

Lipitor is prescribed for a patient with a high cholesterol level. As the nurse, how do you educate the patient on how this drugs works on the body? A. Lipitor increases LDL levels and decreases HDL levels, total cholesterol, and triglyceride levels. B. Lipitor decreases LDL, HDL levels, total cholesterol, and triglyceride levels. C. Lipitor increases HDL levels, total cholesterol, and triglyceride levels. D. Lipitor increases HDL levels and decreases LDL, total cholesterol, and triglyceride levels.

Answer: D. Lipitor is a common "statin" medication used to lower cholesterol in CAD. It works by increasing HDL levels (the "good" cholesterol") and decreases LDL (the "bad" cholesterol"), total cholesterol, and triglyceride levels.

A patient who has diabetes will be started on Metoprolol for medical management of coronary artery disease. Which of the following will you include in your discharge teaching about this medication? A. Check your heart rate regularly because Metoprolol can cause an irregular heart rate. B. Check your glucose regularly because this medication can cause hyperglycemia. C. Check your blood pressure regularly because this medication can cause hypertension. D. Check your glucose regularly because this medication can mask the typical signs and symptoms of hypoglycemia.

Answer: D. This patient needs to be educated to check their glucose levels regularly because this medication (Metoprol) can mask the typical signs and symptoms of hypoglycemia. This is very important since the patient is diabetic.

A patient reports during a routine check-up that he is experiencing chest pain and shortness of breath while performing activities. He states the pain goes away when he rests. What type of diagnostic tests will the physician most likely order (at first) for this patient to evaluate the cause of the patient's symptoms? Select-all-that-apply: A. EKG B. Stress test C. Heart catheterization D. Balloon angioplasty

Answers: A and B. If the patient is experiencing STABLE angina an EKG or stress test would be ordered to investigate if there are any EKG changes (ST depression) during exercise. These tests are usually ordered first and then the doctor may proceed with a heart catheterization. A balloon angioplasty is sometimes performed during a heart cath.

A patient is receiving treatment for stable coronary artery disease. The doctor prescribes the patient Plavix. What important information will you include in the patient's teaching? Select-all-that-apply: A. If you are scheduled for any planned surgical procedures, let your doctor know you are taking Plavix because this medication will need to be discontinued 5-7 days prior to the procedure. B. A normal side effect of this medication is a dry cough. C. Avoid green leafy vegetables while taking Plavix. D. Notify the doctor, immediately, if you develop bruising, problems urinating, or fever.

Answers: A and D. Patients on Plavix should let their doctor know that they are taking Plavix because it should be discontinued 5 to 7 days before a surgical procedure due to increased risk of bleeding. Also, option D represents signs and symptoms of Thrombotic Thrombocytopenic Purpura a clotting disorder where clots form in the vessels of the body which is a complication of Plavix.

A patient admitted with heart failure is anxious and reports shortness of breath. Which nursing actions would be appropriate to alleviate this patient's anxiety? (Select all that apply.) A. Administer ordered morphine sulfate. B. Position patient in a semi-Fowler's position. C. Position patient on left side with head of bed flat. D. Instruct patient on the use of relaxation techniques. E. Use a calm, reassuring approach while talking to patient.

Answers: A, B, D, E Rationale: Morphine sulfate reduces anxiety and may assist in reducing dyspnea. The patient should be positioned in semi-Fowler's position to improve ventilation that will reduce anxiety. Relaxation techniques and a calm reassuring approach will also serve to reduce anxiety.

You're providing education to a patient who will be undergoing a heart catheterization. Which statement by the patient requires you to re-educate the patient about this procedure? A. "The brachial artery is most commonly used for this procedure." B. "A dye is injected into the coronary arteries to assess for blockages." C. "Not all patients who have a heart catheterization will need a stent placement." D. "I will not be completely asleep and will be able to breathe on my own during the procedure."

Answers: A. The femoral or radial artery is used during a heart cath...not the brachial.

Which patient(s) are most at risk for developing coronary artery disease? Select-all-that-apply: A. A 25 year old patient who exercises 3 times per week for 30 minutes a day and has a history of cervical cancer. B. A 35 year old male with a BMI of 30 and reports smoking 2 packs of cigarettes a day. C. A 45 year old female that reports her father died at the age of 42 from a myocardial infraction. D. A 29 year old that has type I diabetes.

Answers: B,C, D. Remember risk factors for developing CAD include: smoking, family history, diabetes, being overweight or obese, and high cholesterol.

The nurse would recognize which assessment finding as suggestive of sepsis? A. Sudden diuresis unrelated to drug therapy B. Hyperglycemia in the absence of diabetes C. Respiratory rate of seven breaths per minute D. Bradycardia with sudden increase in blood pressure

B Hyperglycemia in patients with no history of diabetes is a diagnostic criterion for sepsis. Oliguria, not diuresis, typically accompanies sepsis along with tachypnea and tachycardia.

A patient is admitted to the emergency department vomiting bright red blood. The patient's vital signs are BP of 78/58 mm Hg, pulse of 124 beats/min, respirations of 28 breaths/min, and temperature of 97.2° F (36.2° C). Which provider order should the nurse complete first? A. Obtain a 12-lead ECG and arterial blood gases. B. Rapidly administer 1000 mL normal saline solution IV. C. Start norepinephrine (Levophed) by continuous IV infusion. D. Insert a nasogastric tube and an indwelling bladder catheter.

B Isotonic crystalloids, such as normal saline solution, should be used in the initial resuscitation of hypovolemic shock. Vasopressor drugs (e.g., norepinephrine) may be considered if the patient does not respond to fluid resuscitation and blood products. Other orders (e.g., insertion of nasogastric tube and indwelling bladder catheter and obtaining the diagnostic studies) can be done after fluid resuscitation is started.

The home care nurse visits a patient with chronic heart failure. Which assessment findings would indicate acute decompensated heart failure (pulmonary edema)? A. Fatigue, orthopnea, and dependent edema B. Severe dyspnea and blood-streaked, frothy sputum C. Temperature is 100.4° F and pulse is 102 beats/min D. Respirations 26 breaths/min despite oxygen by nasal cannula

B Manifestations of pulmonary edema include anxiety, pallor, cyanosis, clammy and cold skin, severe dyspnea, use of accessory muscles of respiration, a respiratory rate greater than 30 breaths/min, orthopnea, wheezing, and coughing with the production of frothy, blood-tinged sputum. Auscultation of the lungs may reveal crackles, wheezes, and rhonchi throughout the lungs. The heart rate is rapid, and blood pressure may be elevated or decreased.

Evidenced Based approaches to prevent Ventilator-Associated Pneumonia (VAP) includes: SATA A. Elevate HOB 15 degrees B. Daily sedation vacation and assessment of readiness to extubate C. Daily oral care using chlorhexidine solution D. Proton pump inhibitor or Histamine-2 antagonist E. Anticoagulants and/or sequential compression devices (SCDs)

B, C, D, E We want HOB elevated 45 degrees (exception: high intracranial pressure pts). Pantoprazole (protonix) or Famotidine prevent gastric ulcers (correlations between gastric ulcers and VAP)

The nurse is creating a care plan for a client hospitalized for treatment of cellulitis. The cellulitis does not seem to be responding to the antibiotic therapy. Which risk requiring monitoring secondary to this issue should the nurse include in the care​ plan? (Select all that​ apply.) A. Seizures B. Arthritis C. Serious systemic infection D. Renal failure E. Osteomyelitis

B, C, E

The following evidence-based tehniques may be used to prevent CAUTIs: SATA a. provide twice daily perineal care using chlorhexidine b. keep collection bag below level of bladder c. empty collection bag only when it is full d. collect urine for lab specimens from sample port, regardless of catheter insertion date e. anchor catheter tubing to patient's leg/abdomen

B, E Perineal care should be done with soap and water only. Drain collection bag more, per agency policy (q 4 h). New evidence sugests replacing an indwelling catheter that was place >72 hours if a urine specimen is required.

The nurse collects a drainage sample to be cultured from the affected area of a client with cellulitis. Which organism should the nurse suspect is the most likely cause of the​ cellulitis? A. Escherichia coli B. Staphylococcus aureus C. Bacillus subtilis D. Group A Streptococcus

B. Staphylococcus aureus

A patient with a recent diagnosis of heart failure has been prescribed furosemide (Lasix). What outcome would demonstrate medication effectiveness? A. Promote vasodilation. B. Reduction of preload. C. Decrease in afterload. Increase in contractility.

B. reduction in preload A. Diuretics such as furosemide are used in the treatment of heart failure to mobilize edematous fluid, reduce pulmonary venous pressure, and reduce preload. They do not directly influence afterload, contractility, or vessel tone.

A patient's localized infection has become systemic and septic shock is suspected. What medication would be given to treat septic shock refractory to fluids? A. Insulin infusion B. Furosemide IV push C. Norepinephrine administered by titration D. Administration of nitrates and β-adrenergic blockers

C If fluid resuscitation using crystalloids is not effective, vasopressor medications, such as norepinephrine (Levophed) and dopamine, are indicated to restore mean arterial pressure (MAP). Nitrates and β-adrenergic blockers are most often used in the treatment of patients in cardiogenic shock. Furosemide (Lasix) is indicated for patients with fluid volume overload. Insulin infusion may be given to normalize blood sugar and improve overall outcomes, but it is not considered a medication used to treat shock.

The nurse is administering a dose of Digitalis (digoxin) to a patient with heart failure (HF). The nurse would become concerned with the possibility of digitalis toxicity if the patient reported which symptom? A. Muscle aches B. Constipation C. Loss of appetite D. Pounding headache

C Anorexia, nausea, vomiting, blurred or yellow vision, and dysrhythmias are all signs of digitalis toxicity. The nurse would become concerned and notify the health care provider if the patient exhibited any of these symptoms.

After having a myocardial infarction (MI), the nurse notes the patient has jugular venous distention, gained weight, developed peripheral edema, and has a heart rate of 108 beats/min. What should the nurse suspect is happening? A. Chronic HF B. Left-sided HF C. Right-sided HF D. Acute decompensated HF

C. An MI is a primary cause of heart failure. The jugular venous distention, weight gain, peripheral edema, and increased heart rate are manifestations of right-sided heart failure.

Which diagnostic test will be most useful to the nurse in determining whether a patient admitted with acute shortness of breath has heart failure? A. Serum troponin B. Arterial blood gases C. B-type natriuretic peptide D. 12-lead electrocardiogram

C. BNP is a marker of cardiac stress/ work overload/ abnormal stretching of heart muscle

The nurse is providing home care instruction to the client with cellulitis. Which​ statement, if made by the​ client, should concern the​ nurse? A. ​"I will keep all​ follow-up appointments with my healthcare​ provider." B. ​"I will be sure to get enough rest and stay off my affected​ leg." C. ​"I will take my antibiotics until the affected area looks less​ red." D. ​"I will keep my affected leg elevated to keep swelling​ down."

C. ​"I will take my antibiotics until the affected area looks less​ red."

Which family of drugs are the following medications considered: Amlodipine, Verapamil, Diltiazem? a. Beta blockers (BB) (-olols) b. ACE Inhibitors (ACEI) (-prils) c. Angiotension Receptor Blockers (ARBs) (-sartans) d. Calcium Channel Blockers (CCBs)

Calcium Channel Blockers (CCBs)

The nurse is caring for a patient admitted with a history of hypertension. The patient's medication history includes hydrochlorothiazide daily for the past 10 years. Which parameter would indicate the optimal intended effect of this drug therapy? A. BP 128/78 mm Hg B. Weight loss of 2 lb C. Absence of ankle edema D. Output of 600 mL per 8 hours

Correct Answer: BP 128/78 mm Hg Rationale: Hydrochlorothiazide may be used alone as monotherapy to manage hypertension or in combination with other medications if not effective alone. After the first few weeks of therapy, the diuretic effect diminishes, but the antihypertensive effect remains. Because the patient has been taking this medication for 10 years, the most direct measurement of its intended effect would be the blood pressure.

A patient with a history of chronic hypertension is being evaluated in the emergency department for a blood pressure of 200/140 mm Hg. Which patient assessment question is the priority? A. Is the patient pregnant? B. Does the patient need to urinate? C. Does the patient have a headache or confusion? D. Is the patient taking antiseizure medications as prescribed?

Correct Answer: Does the patient have a headache or confusion? Rationale: The nurse's priority assessments include neurologic deficits, retinal damage, heart failure, pulmonary edema, and renal failure. The headache or confusion could be seen with hypertensive encephalopathy from increased cerebral capillary permeability leading to cerebral edema. In addition, headache or confusion could represent signs and symptoms of a hemorrhagic stroke. Pregnancy can lead to secondary hypertension. Needing to urinate and taking antiseizure medication do not support a hypertensive emergency.

When teaching how lisinopril (Zestril) will help lower the patient's blood pressure, which mechanism of action should the nurse explain? A. Blocks β-adrenergic effects. B. Relaxes arterial and venous smooth muscle. C. Inhibits conversion of angiotensin I to angiotensin II. D. Reduces sympathetic outflow from central nervous system.

Correct Answer: Inhibits conversion of angiotensin I to angiotensin II. Rationale: Lisinopril is an angiotensin-converting enzyme inhibitor that inhibits the conversion of angiotensin I to angiotensin II, which reduces angiotensin II-mediated vasoconstriction and sodium and water retention. β-Blockers result in vasodilation and decreased heart rate. Direct vasodilators relax arterial and venous smooth muscle. Central-acting α-adrenergic antagonists reduce sympathetic outflow from the central nervous system to produce vasodilation and decreased systemic vascular resistance and blood pressure.

The nurse is caring for a patient with hypertension who is scheduled to receive a dose of metoprolol (Lopressor). The nurse should withhold the dose and consult the prescribing provider for which vital sign taken just before administration? A. O2 saturation 93% B. Pulse 48 beats/min C. Respirations 24 breaths/min D. Blood pressure 118/74 mm Hg

Correct Answer: Pulse 48 beats/min Rationale: Because metoprolol is a β1-adrenergic blocking agent, it can cause hypotension and bradycardia as adverse effects. The nurse should withhold the dose and consult with the health care provider for parameters regarding pulse rate limits.

When teaching a patient about dietary management of stage 1 hypertension, which instruction is appropriate? A. Increase water intake. B. Restrict sodium intake. C. Increase protein intake. D. Use calcium supplements.

Correct Answer: Restrict sodium intake. Rationale: The patient should decrease intake of sodium. This will help to control hypertension, which can be aggravated by excessive salt intake, which in turn leads to fluid retention. Protein intake does not affect hypertension. Calcium supplements are not recommended to lower blood pressure. JNC: "Reduce sodium intake to no more than 2,400 mg/day (<1,500 mg/day for Black, middle-age, those with HTN/DM/CKD"

In caring for a patient admitted with poorly controlled hypertension, which laboratory test result should the nurse understand as indicating the presence of target organ damage? A. Serum uric acid of 3.8 mg/dL B. Serum creatinine of 2.6 mg/dL C. Serum potassium of 3.5 mEq/L D. Blood urea nitrogen of 15 mg/dL

Correct Answer: Serum creatinine of 2.6 mg/dL Rationale: The normal serum creatinine level is 0.6 to 1.3 mg/dL. This elevated level indicates target organ damage to the kidneys. The other laboratory results are within normal limits. (Harding et al. Med Surg text: Normal Creatinine men: 0.6-1.3; women: 0.5-1.1)

After teaching a patient with chronic stable angina about nitroglycerin, the nurse recognizes the need for further teaching when the patient makes which statement? A. "I will replace my nitroglycerin supply every 6 months." B. "I can take up to 5 tablets every 3 minutes for relief of my chest pain." C. "I will take acetaminophen (Tylenol) to treat the headache caused by nitroglycerin." D. "I will take the nitroglycerin 10 minutes before planned activity that usually causes chest pain."

Correct Answer: "I can take up to 5 tablets every 3 minutes for relief of my chest pain." Rationale: The recommended dose of nitroglycerin is one tablet taken sublingually (SL) or 1 metered spray for symptoms of angina. If symptoms are unchanged or worse after 5 minutes, the patient should be instructed to activate the emergency medical services (EMS) system. If symptoms are improved, repeat the nitroglycerin every 5 minutes for a maximum of 3 doses and contact EMS if symptoms have not resolved completely.

A 44-yr-old man is diagnosed with hypertension and receives a prescription for benazepril (Lotensin). After providing teaching, which statement by the patient indicates correct understanding? A. "If I take this medication, I will not need to follow a special diet." B. "It is normal to have some swelling in my face while taking this medication." C. "I will need to eat foods such as bananas and potatoes that are high in potassium." D. "If I develop a dry cough while taking this medication, I should notify my doctor."

Correct Answer: "If I develop a dry cough while taking this medication, I should notify my doctor." Rationale: Benazepril is an angiotensin-converting enzyme inhibitor. The medication inhibits breakdown of bradykinin, which may cause a dry, hacking cough. Other adverse effects include hyperkalemia. Swelling in the face could indicate angioedema and should be reported immediately to the prescriber. Patients taking drug therapy for hypertension should also attempt lifestyle modifications to lower blood pressure such as a reduced sodium diet.

Which patient is at greatest risk for sudden cardiac death (SCD)? A. A 52-yr-old black man with left ventricular failure B. A 62-yr-old obese man with diabetes and high cholesterol C. A 42-yr-old white woman with hypertension and dyslipidemia D. A 72-yr-old Native American woman with a family history of heart disease

Correct Answer: A 52-yr-old black man with left ventricular failure Rationale: Patients with left ventricular dysfunction (ejection fraction less than 30%) and ventricular dysrhythmias after myocardial infarction are at greatest risk for SCD. Other risk factors for SCD include: (1) male gender (especially blacks), (2) family history of premature atherosclerosis, (3) tobacco use, (4) diabetes, (5) high cholesterol levels, (6) hypertension, and (7) cardiomyopathy.

A patient who had bladder surgery 2 days ago develops acute decompensated heart failure (ADHF) with severe dyspnea. Which action by the nurse would be indicated first? A. Review urinary output for the previous 24 hours. B. Restrict the patient's oral fluid intake to 500 mL/day. C. Assist the patient to a sitting position with arms on the overbed table. D. Teach the patient to use pursed-lip breathing until the dyspnea subsides.

Correct Answer: Assist the patient to a sitting position with arms on the overbed table. Rationale: The nurse should place the patient with ADHF in a high Fowler's position with the feet horizontal in the bed or dangling at the bedside. This position helps decrease venous return because of the pooling of blood in the extremities. This position also increases the thoracic capacity, allowing for improved ventilation. Pursed-lip breathing helps with obstructive air trapping but not with acute pulmonary edema. Restricting fluids takes considerable time to have an effect.

Postoperative care of a patient undergoing coronary artery bypass graft (CABG) surgery includes monitoring for which common complication? A. Dehydration B. Paralytic ileus C. Atrial dysrhythmias D. Acute respiratory distress syndrome

Correct Answer: Atrial dysrhythmias Rationale: Postoperative dysrhythmias, specifically atrial dysrhythmias, are common in the first 3 days after CABG surgery. Although the other complications could occur, they are not common complications.

The nurse is preparing to administer a nitroglycerin patch to a patient. When providing teaching about the use of the patch, what should the nurse include? A. Avoid drugs to treat erectile dysfunction. B. Increase diet intake of high-potassium foods. C. Take an over-the-counter H2-receptor blocker. D. Avoid nonsteroidal antiinflammatory drugs (NSAIDS).

Correct Answer: Avoid drugs to treat erectile dysfunction. Rationale: The use of erectile drugs concurrent with nitrates creates a risk of severe hypotension and possibly death. NSAIDs do not pose a risk in combination with nitrates. There is no need to take an H2-receptor blocker or increase the dietary intake of high-potassium foods.

The nurse is caring for a patient admitted with chronic obstructive pulmonary disease (COPD), angina, and hypertension. Before administering the prescribed daily dose of atenolol 100 mg PO, the nurse assesses the patient carefully. Which adverse effect is this patient at risk for given the patient's health history? A. Hypocapnia B. Tachycardia C. Bronchospasm D. Nausea and vomiting

Correct Answer: Bronchospasm Rationale: Atenolol is a cardioselective β1-adrenergic blocker that reduces blood pressure and could affect the β2-receptors in the lungs with larger doses or with drug accumulation. Although the risk of bronchospasm is less with cardioselective β-blockers than nonselective β-blockers, atenolol should be used cautiously in patients with COPD.

The nurse admits a 73-yr-old male patient with dementia for treatment of uncontrolled hypertension. The nurse will closely monitor for hypokalemia if the patient receives which medication? A. Clonidine (Catapres) B. Bumetanide (Bumex) C. Amiloride (Midamor) D. Spironolactone (Aldactone)

Correct Answer: Bumetanide (Bumex) Rationale: Bumetanide is a loop diuretic. Hypokalemia is a common adverse effect of this medication. Amiloride is a potassium-sparing diuretic. Spironolactone is an aldosterone-receptor blocker. Hyperkalemia is an adverse effect of both amiloride and spironolactone. Clonidine is a central-acting α-adrenergic antagonist and does not cause electrolyte abnormalities.

A patient admitted to the emergency department 24 hours ago with reports of chest pain was diagnosed with a ST-segment-elevation myocardial infarction (STEMI). What complication of myocardial infarction should the nurse anticipate? A. Dysrhythmias B. Unstable angina C. Cardiac tamponade D. Sudden cardiac death

Correct Answer: Dysrhythmias Rationale: Dysrhythmias are present in 80% to 90% of patients after myocardial infarction (MI). Unstable angina is considered a precursor to MI rather than a complication. Cardiac tamponade is a rare event, and sudden cardiac death is defined as an unexpected death from cardiac causes. Cardiac dysfunction in the period following an MI would not be characterized as sudden cardiac death.

Which assessment finding should be considered when caring for a woman with suspected coronary artery disease? A. Fatigue may be the first symptom. B. Classic signs and symptoms are expected. C. Increased risk is present before menopause. D. Women are more likely to develop collateral circulation.

Correct Answer: Fatigue may be the first symptom. Rationale: Fatigue, rather than pain or shortness of breath, may be the first symptom of impaired cardiac circulation. Women may not exhibit the classic signs and symptoms of ischemia such as chest pain which radiates down the left arm. Neck, throat, or back pain may be symptoms experienced by women. Risk for coronary artery disease increases four times after menopause. Men are more likely to develop collateral circulation.

The nurse teaches a patient with hypertension that uncontrolled hypertension may damage organs in the body primarily by which mechanism? A. Hypertension promotes atherosclerosis and damage to the walls of the arteries. B. Hypertension causes direct pressure on organs, resulting in necrosis and scar tissue. C. Hypertension causes thickening of the capillary membranes, leading to hypoxia of organ systems. Hypertension increases blood viscosity

Correct Answer: Hypertension promotes atherosclerosis and damage to the walls of the arteries. Rationale: Hypertension is a major risk factor for the development of atherosclerosis by mechanisms not yet fully known. However, when atherosclerosis develops, it damages the walls of arteries and reduces circulation to target organs and tissues.

The nurse in the recovery room assesses the right femoral artery puncture site after the patient had a stent inserted into a coronary artery. The insertion site is not bleeding or discolored. What should the nurse do next to ensure the femoral artery is intact? A. Palpate the insertion site for induration. B. Assess peripheral pulses in the right leg. C. Inspect the patient's right side and back. D. Compare the color of the left and right legs.

Correct Answer: Inspect the patient's right side and back. Rationale: The best method to determine that the right femoral artery is intact after inspection of the insertion site is to logroll the patient to inspect the right side and back for retroperitoneal bleeding. The artery can be leaking, and blood is drawn into the tissues by gravity. The peripheral pulses, color, and sensation of the right leg will be assessed per agency protocol.

The nurse is caring for a patient who was admitted 1 week ago with multiple rib fractures, pulmonary contusions, and a left femur fracture from a motor vehicle crash. The provider states the patient has developed sepsis, and the family members have many questions. Which information should the nurse include when explaining the early stage of sepsis? A. Weaning the patient from the ventilator is the top priority in sepsis. B. Antibiotics are not useful when an infection has progressed to sepsis. C. Large amounts of IV fluid are required in sepsis to fill dilated blood vessels. D. The patient has recovered from sepsis if he has warm skin and ruddy cheeks.

Correct Answer: Large amounts of IV fluid are required in sepsis to fill dilated blood vessels. Rationale: Patients with sepsis may be normovolemic, but because of acute vasodilation, relative hypovolemia and hypotension occur. Patients in septic shock require large amounts of fluid replacement and may require frequent fluid boluses to maintain circulation. Antibiotics are an important component of therapy for patients with septic shock. They should be started after cultures (e.g., blood, urine) are obtained and within the first hour of septic shock. Oxygenating the tissues is the top priority in sepsis, so efforts to wean septic patients from mechanical ventilation halt until sepsis is resolving. Additional respiratory support may be needed during sepsis. Although cool and clammy skin is present in other early shock states, the patient in early septic shock may feel warm and flushed because of a hyperdynamic state.

A 74-yr-old man with a history of prostate cancer and hypertension is admitted to the emergency department with substernal chest pain. Which priority action will the nurse complete before administering sublingual nitroglycerin? A. Administer morphine sulfate IV. B. Auscultate heart and lung sounds. C. Obtain a 12-lead electrocardiogram (ECG). D. Assess for coronary artery disease risk factors.

Correct Answer: Obtain a 12-lead electrocardiogram (ECG). Rationale: If a patient has chest pain, the nurse should institute the following measures: (1) administer supplemental oxygen and position the patient in upright position unless contraindicated, (2) assess vital signs, (3) obtain a 12-lead ECG, (4) provide prompt pain relief first with a nitrate followed by an opioid analgesic if needed, and (5) auscultate heart sounds. Obtaining a 12-lead ECG during chest pain aids in the diagnosis.

The nurse is examining the electrocardiogram (ECG) of a patient just admitted with a suspected myocardial infarction (MI). Which ECG change is most indicative of prolonged or complete coronary occlusion? A. Sinus tachycardia B. Pathologic Q wave C. Fibrillatory P waves D. Prolonged PR interval

Correct Answer: Pathologic Q wave Rationale: The presence of a pathologic Q wave, as often accompanies STEMI, is indicative of complete coronary occlusion. Sinus tachycardia, fibrillatory P waves (e.g., atrial fibrillation), or a prolonged PR interval (first-degree heart block) are not direct indicators of extensive occlusion.

A patient has received a bolus dose and an infusion of alteplase (Activase) for an ST-segment elevation myocardial infarction (STEMI). Which patient assessment would determine the effectiveness of the medication? A. Presence of chest pain B. Blood in the urine or stool C. Tachycardia with hypotension D. Decreased level of consciousness

Correct Answer: Presence of chest pain Rationale: Alteplase is a fibrinolytic agent that is administered to patients who have had a STEMI. If the medication is effective, the patient's chest pain will resolve because the medication dissolves the thrombus in the coronary artery and results in reperfusion of the myocardium. Bleeding is a major complication of fibrinolytic therapy. Signs of major bleeding include decreased level of consciousness, blood in the urine or stool, and increased heart rate with decreased blood pressure.

A 67-yr-old woman with hypertension is admitted to the emergency department with a blood pressure of 234/148 mm Hg and was started on nitroprusside (Nitropress). After 1 hour of treatment, the mean arterial blood pressure (MAP) is 55 mm Hg. Which nursing action is a priority? A. Start an infusion of 0.9% normal saline at 100 mL/hr. B. Maintain the current administration rate of the nitroprusside. C. Request insertion of an arterial line for accurate blood pressure monitoring. D. Stop the nitroprusside infusion and assess the patient for potential complications

Correct Answer: Stop the nitroprusside infusion and assess the patient for potential complications. Rationale: Nitroprusside is a potent vasodilator medication. A blood pressure of 234/118 mm Hg would have a calculated MAP of 177 mm Hg. Subtracting 25% (or 44 mm Hg) = 133 mm Hg. The initial treatment goal is to decrease MAP by no more than 25% within minutes to 1 hour. For this patient, the goal MAP would be around 133 mm Hg. Minimal MAP required to perfuse organs is around 60 to 65 mm Hg. Lowering the blood pressure too rapidly may decrease cerebral, coronary, or renal perfusion and could precipitate a stroke, myocardial infarction, or renal failure. The priority is to stop the nitroprusside infusion and then use fluids only if necessary to support restoration of MAP.

A male patient with coronary artery disease (CAD) has a low-density lipoprotein (LDL) cholesterol of 98 mg/dL and high-density lipoprotein (HDL) cholesterol of 47 mg/dL. What information should the nurse include in patient teaching? A. Consume a diet low in fats. B. Reduce total caloric intake. C. Increase intake of olive oil. D. The lipid levels are normal

Correct Answer: The lipid levels are normal. Rationale: For men, the recommended LDL is less than 100 mg/dL, and the recommended level for HDL is greater than 40mg/dL. His normal lipid levels should be included in the patient teaching and encourage him to continue taking care of himself. Assessing his need for teaching related to diet should also be done.

When a person's blood pressure rises, the homeostatic mechanism to compensate for an elevation involves stimulation of a. baroreceptors that inhibit the sympathetic nervous system, causing vasodilation. b. chemoreceptors that inhibit the sympathetic nervous system, causing vasodilation. c. baroreceptors that inhibit the parasympathetic nervous system, causing vasodilation. d. chemoreceptors that stimulate the sympathetic nervous system, causing an increased heart rate.

Correct answer: a Rationale: Baroreceptors in the aortic arch and carotid sinus are sensitive to stretch or pressure within the arterial system. Stimulation of these receptors sends information to the vasomotor center in the brainstem. This results in temporary inhibition of the sympathetic nervous system and enhancement of the parasympathetic influence, which cause a decrease in heart rate and peripheral vasodilation.

A patient with newly discovered high BP has an average reading of 158/98 mm Hg after 3 months of exercise and diet modifications. Which management strategy will be a priority for this patient? a. Drug therapy will be needed because the BP is still not at goal. b. BP monitoring should continue for 3 months to confirm a diagnosis of hypertension. c. Lifestyle changes are less important since they were not effective, and drugs will be started. d. More changes in the patient's lifestyle are needed for a longer time before starting drug therapy.

Correct answer: a Rationale: The patient has hypertension, stage 1. Lifestyle modifications will continue, but starting drug therapy is a priority. Reducing BP can help to prevent serious complications related to hypertension.

The nurse is caring for a patient who is 2 days post MI. The patient reports that she is experiencing chest pain when she takes a deep breath. Which action would be a priority? a. Notify the provider STAT and obtain a 12-lead ECG. b. Obtain vital signs and auscultate for a pericardial friction rub. c. Apply high-flow O2 by face mask and auscultate breath sounds. d. Medicate the patient with as-needed analgesic and reevaluate in 30 minutes.

Correct answer: b Rationale: Acute pericarditis is inflammation of the visceral and/or parietal pericardium. It often occurs 2 to 3 days after an acute myocardial infarction. Chest pain may vary from mild to severe. It is worsened by inspiration, coughing, and movement of the upper body. Sitting in a forward position often relieves the pain. The pain is usually different from pain associated with a myocardial infarction. Assessment of the patient with pericarditis may reveal a friction rub over the pericardium.

While obtaining subjective assessment data from a patient with hypertension, the nurse recognizes that a modifiable risk factor for the development of hypertension is a. A low-calcium diet. b. Excess alcohol intake. c. A family history of hypertension. d. Consumption of a high-protein diet.

Correct answer: b Rationale: Alcohol intake is a modifiable risk factor for hypertension. Excessive alcohol intake is strongly associated with hypertension. Males with hypertension should limit their daily intake of alcohol to 2 drinks per day, and one drink per day for females with hypertension.

A hospitalized patient with a history of chronic stable angina tells the nurse that she is having chest pain. The nurse bases his actions on the knowledge that ischemia a. will always progress to myocardial infarction. b. can be relieved by rest, nitroglycerin, or both. c. is often associated with vomiting and extreme fatigue. d. indicates that irreversible myocardial damage is occurring.

Correct answer: b Rationale: Chronic stable angina is chest pain that occurs intermittently over a long period with the same pattern of onset, duration, and intensity of symptoms. The chest pain is relieved by rest or by rest and medication (e.g., nitroglycerin). The ischemia is transient and does not cause myocardial damage.

What compensatory mechanism involved in both chronic heart failure and acute decompensated heart failure leads to fluid retention and edema? a. Ventricular dilation b. Ventricular hypertrophy c. Increased systemic blood pressure d. Renin-angiotensin-aldosterone activation

Correct answer: d Rationale: Compensatory mechanisms in both chronic and acute HF include fluid and sodium retention and edema resulting from activation of the renin angiotensin aldosterone system (RAAS). Ventricular dilation, hypertrophy, and high BP do not cause fluid retention and edema.

A patient with chronic heart failure and atrial fibrillation is treated with low-dose digitalis and a loop diuretic. What does the nurse need to do to prevent complications of this drug combination? (select all that apply) a. Monitor serum potassium levels. b. Teach the patient how to take a pulse rate. c. Withhold digitalis if pulse rhythm is irregular. d. Keep an accurate measure of intake and output. e. Teach the patient about dietary potassium restrictions.

Correct answers: a, b Rationale: Hypokalemia, which can be caused by using potassium-depleting diuretics (e.g., thiazides, loop diuretics), is a common cause of digitalis toxicity. Low serum levels of potassium enhance the action of digitalis, causing a therapeutic dose to achieve toxic levels. Hypokalemia can precipitate dysrhythmias. Monitoring the serum potassium levels of patients receiving digitalis preparations and potassium-depleting diuretics is essential. Teach patients taking digitalis preparations how to measure their pulse rate because bradycardia and atrioventricular blocks are late signs of digitalis toxicity. Patients should know what pulse rate would require a call to the HCP. Patients should not independently decide to skip a dose of digitalis.

Which nursing responsibilities are priorities when caring for a patient returning from a cardiac catheterization (select all that apply)? a. Monitoring vital signs and ECG b. Checking the catheter insertion site and distal pulses c. Helping the patient to ambulate to the bathroom to void d. Telling the patient that he will be sleepy from the general anesthesia e. Teaching the patient about the risks of the radioactive isotope injection

Correct answers: a, b Rationale: The nursing responsibilities after cardiac catheterization include assessing the puncture site for hematoma and bleeding; assessing circulation to the extremity used for catheter insertion for peripheral pulses, color, and sensation; and monitoring vital signs and electrocardiographic rhythm.

Which statements accurately describe heart failure with preserved ejection fraction (HFpEF)? (select all that apply) a. Uncontrolled hypertension is the primary cause. b. Left ventricular ejection fraction may be within normal limits. c. The pathophysiology involves ventricular relaxation and filling. d. Multiple evidence-based therapies have been shown to decrease mortality. e. Therapies focus on symptom control and treatment of underlying conditions.

Correct answers: a, b, c, e Rationale: Uncontrolled HTN is the main cause. HFpEF results from impaired ventricular relaxation and filling and LVEF may be normal. Therapies concentrate on symptom management and treatment of underlying conditions. To date, there are no therapies shown to decrease mortality or morbidity in patients with HFpEF.

A patient has a severe blockage in his right coronary artery. Which heart structures are most likely to be affected by this blockage (select all that apply)? a. AV node b. Left ventricle c. Coronary sinus d. Right ventricle e. Pulmonic valve

Correct answers: a, b, d Rationale: The right coronary artery (RCA) supplies blood to the right atrium, the right ventricle, and part of the posterior wall of the left ventricle. In 90% of people, the RCA supplies blood to the atrioventricular (AV) node, bundle of His, and part of the cardiac conduction system.

A patient is admitted to the hospital in a hypertensive emergency (BP 244/142 mm Hg). Sodium nitroprusside is started to treat the elevated BP. Which management strategies would be most appropriate for this patient? (select all that apply) a. Measuring hourly urine output b. Continuous BP monitoring with an arterial line c. Decreasing the MAP by 50% within the first hour d. Maintaining bed rest and giving tranquilizers to lower the BP e. Assessing the patient for signs and symptoms of heart failure and changes in mental status

Correct answers: a, b, e Rationale: Measure urine output hourly to assess renal perfusion. Patients treated with IV sodium nitroprusside should have continuous intraarterial BP monitoring. Hypertensive crisis can cause encephalopathy, intracranial or subarachnoid hemorrhage, acute left ventricular failure, MI, renal failure, dissecting aortic aneurysm, and retinopathy. The initial treatment goal is to decrease the mean arterial pressure (MAP) by no more than 25% within minutes to 1 hour. Patients receiving IV antihypertensive drugs may be restricted to bed rest. Getting up (e.g., to use the toilet/commode) may cause severe cerebral ischemia and fainting. ****Hypertensive emergency is a SBP >180 mmHg and/or DBP >110 mmHg + evidence of target organ damage (heart, brain, kidneys, and eyes, and PVS are most at risk)

Which BP-regulating mechanism(s) can result in the development of hypertension if defective? (select all that apply) a. Release of norepinephrine b. Secretion of prostaglandins c. Stimulation of the sympathetic nervous system d. Stimulation of the parasympathetic nervous system e. Activation of the renin-angiotensin-aldosterone system

Correct answers: a, c, e Rationale: Norepinephrine (NE) is released from the sympathetic nervous system nerve endings and activates receptors found in the vascular smooth muscle. When the α-adrenergic receptors in smooth muscle of the blood vessels are stimulated by NE, vasoconstriction results. Increased sympathetic nervous system stimulation produces increased vasoconstriction and increased renin release. Increased renin levels activate the renin-angiotensin-aldosterone system, leading to an elevation in BP.

Interventions to prevent health care-associated infections include (select all that apply) a. following hand-washing protocols. b. limiting visitors to persons over age 18. c. placing high-risk patients in private rooms. d. decontaminating equipment used for patient care. e. appropriately using personal protective equipment.

Correct answers: a, d, e Rationale: First lines of defense to prevent the spread of HAIs include hand washing (or using an alcohol-based hand sanitizer) before and after patient contact or procedures, appropriate use of personal protective equipment such as gloves, and decontamination of equipment used for patient care.

The nurse is caring for a patient with acute decompensated heart failure who is receiving IV dobutamine. Why would this drug be prescribed? (select all that apply) a. It dilates renal blood vessels. b. It will increase the heart rate. c. Heart contractility will improve. d. Dobutamine is a selective β-agonist. e. It increases systemic vascular resistance.

Correct answers: c, d Rationale: Dobutamine has a positive inotropic effect (improves contractility). It is used for patients who, despite conventional therapies, have a very low CO. It is a selective β-adrenergic agonist and works primarily on the β1-adrenergic receptors in the heart. It does not increase heart rate or systemic vascular resistance. Dopamine, not dobutamine, dilates renal blood vessels.

The nurse would assess a patient with reports of chest pain for which clinical manifestations associated with a myocardial infarction (MI)? (Select all that apply.) A. Flushing B. Ashen skin C. Diaphoresis D. Nausea and vomiting E. S3 or S4 heart sounds

Correct: B, C, D, E Rationale: During the initial phase of an MI, catecholamines are released from the ischemic myocardial cells, causing increased sympathetic nervous system stimulation. This results in the release of glycogen, diaphoresis, and vasoconstriction of peripheral blood vessels. The patient's skin may be ashen, cool, and clammy (not flushed) because of this response. Nausea and vomiting may result from reflex stimulation of the vomiting center by severe pain. Ventricular dysfunction resulting from the MI may lead to the presence of the abnormal S3 and S4 heart sounds.

The most common finding in people at risk for sudden cardiac death is a. aortic valve disease. b. mitral valve disease. c. left ventricular dysfunction. d. atherosclerotic heart disease.

Correct: C Rationale: Left ventricular dysfunction (ejection fraction less than 30%) and ventricular dysrhythmias after myocardial infarction are the strongest predictors of sudden cardiac death (SCD).

A patient on the cardiac critical care unit had a myocardial infarction 2 days ago. The nurse notes the following vital signs: HR 115, BP 148/97, SpO2 89% on 6L NC, RR 30, temperature 38.0. Which vital sign is an expected finding following a myocardial infarction and does not require follow-up with the provider? A. Heart rate Blood pressure Pulse oximetry D. Temperature

D. The temperature may increase to 100.4° F (38° C) within the first 24-48 hours following an Ml. The temperature elevation may last for as long as 4-5 days. This increase in temperature is due to a systemic inflammatory process caused by the death of heart cells.

What laboratory finding is consistent with cardiogenic shock? A. Decreased liver enzymes B. Increased white blood cells C. Decreased red blood cells, hemoglobin, and hematocrit D. Increased blood urea nitrogen (BUN) and creatinine levels

D. Increased blood urea nitrogen (BUN) and creatinine levels The renal hypoperfusion that accompanies cardiogenic shock results in increased BUN and creatinine levels. Impaired perfusion of the liver results in increased liver enzymes, but white blood cell levels do not typically increase in cardiogenic shock. Red blood cell indices are typically normal because of relative hypovolemia.

The nurse examines a wound on a client with a history of cellulitis. Which manifestation suggests​ cellulitis? A. Intact skin with nonblanchable redness and elevated borders B. Reddened skin with indistinct borders and covered by a​ yellow, fibrous film C. Pink or red skin with circumscribed regular borders D. Red or lilac edematous skin with a​ well-defined, nonelevated border

D. Red or lilac edematous skin with a​ well-defined, nonelevated border

A patient is being discharged home after hospitalization of left ventricular systolic dysfunction. As the nurse providing discharge teaching to the patient, which statement is NOT a correct statement about this condition? A. "Signs and symptoms of this type of heart failure can include: dyspnea, persistent cough, difficulty breathing while lying down, and weight gain." B. "It is important to monitor your daily weights, fluid and salt intake." C. "Left-sided heart failure can lead to right-sided heart failure, if left untreated." D. "This type of heart failure can build up pressure in the hepatic veins and cause them to become congested with fluid which leads to peripheral edema."

Option D is the answer. This is a description of right-sided heart failure NOT left ventricular systolic dysfunction. Left-sided systolic dysfunction is where the left side of the heart is unable to CONTRACT efficiently which causes blood to back-up into the lungs...leading to pulmonary edema.

Which drug prescribed to a client with a urinary tract infection (UTI) turns urine reddish-orange in color? · Ciprofloxacin · Nitrofurantoin · Phenazopyridine · Amoxicillin

Phenazopyridine is a topical anesthetic that is used to treat pain or burning sensation associated with urination. It also imparts a characteristic orange or red color to urine. Amoxicillin is a penicillin form that could cause pseudomembranous colitis as a complication; it is not associated with reddish-orange colored urine. Ciprofloxacin is a quinolone antibiotic used for treating UTIs and can cause serious cardiac dysrhythmias and sunburns. It is not, however, responsible for reddish-orange colored urine. Nitrofurantoin is an antimicrobial medication prescribed for UTIs. This drug may affect the kidneys but is not associated with reddish-orange colored urine.

The nurse prepares to administer digoxin 0.125 mg to a patient admitted with influenza and a history of chronic heart failure. What should the nurse assess before giving the medication? A. Prothrombin time B. Urine specific gravity C. Serum potassium level D. Hemoglobin and hematocrit

Serum potassium should be monitored because hypokalemia increases the risk for digoxin toxicity. Changes in prothrombin time, urine specific gravity, and hemoglobin or hematocrit would not require holding the digoxin dose.

What class of drug inhibits the synthesis of cholesterol in the liver, in which rhabdomyolysis is an uncommon side effect?

Statins (e.g., simvastatin)

True or False:Community Acquired MRSA (CA-MRSA) is more virulent than Hospital-Acquired MRSA

TRUE

A patient is receiving treatment for an acute myocardial infarction. The nurse is closely monitoring the patient for signs and symptoms associated with cardiogenic shock. Which value below is associated with cardiogenic shock? A. Cardiac index 1.5 L/min/m2 B. Pulmonary capillary wedge pressure (PCWP) 10 mmHg C. Central venous pressure (CVP) 4 mmHg D. Troponin <0.01 ng/mL

The answer is A. A patient who has experienced an acute myocardial infarction is at risk for cardiogenic shock. A normal cardiac index should be 2.5-4 L/min/m2. Cardiac index is the specific cardiac output based on the patient's size. Any number less than 2.2 L/min/m2 can be associated with cardiogenic shock. All the other values are within normal limits. PCWP would be >18 mmHg in cardiogenic shock, and CVP would be high.

A 76 year old female is admitted due to a recent fall. The patient is confused and agitated. The family members report that this is not normal behavior for the patient. They explain that the patient is very active in the community and cares for herself. Based on the information you have gathered about the patient, which physician's order takes priority? A. "Collect a urinalysis" B. "Collect a T3 and T4 level" C. "Insert a Foley Catheter" D. "Keep patient NPO"

The answer is A. Elderly patients do NOT exhibit the typical signs and symptoms of a UTI. Instead, they may become confused, experience falls, become agitated etc. This can occurs in elderly patients who are normally alert and oriented and active. If this is seen in your patient, think UTI. Collecting a urinalysis (per MD order) is very important to help determine the sudden cause of confused/agitation and falling. If the U/A comes back positive for WBCs and bacteria the patient can start receiving the proper treatment.

You're caring for a patient with cardiogenic shock. Which finding below suggests the patient's condition is worsening? Select all that apply: A. Blood pressure 95/68 B. Urinary output 20 mL/hr C. Cardiac Index 3.2 L/min/m2 D. Pulmonary artery wedge pressure 30 mmHg

The answer is B and D. When answering this question look for values that are abnormal and that point to worsening tissue perfusion (urinary output should be 30 mL/hr or greater....if it's lower than this it show the kidneys are not being perfused) and worsening cardiac output (the blood pressure and cardiac index are within normal limits BUT pulmonary artery wedge pressure is NOT). A pulmonary artery wedge pressure (also called pulmonary capillary wedge pressure) is the pressure reading of the filling pressure in the left atrium. A normal PAWP is 4-12 mmHg and if it's >18 mmHg this indicates cardiogenic shock. If it reads high, that means there is back-flowing of blood into the heart and lungs (hence the left ventricle is failing to pump efficiently and increasing the pressure in the left atrium).

Which statement below best describes the term cardiac preload? A. The pressure the ventricles stretch at the end of systole. B. The amount the ventricles stretch at the end of diastole. C. The pressure the ventricles must work against to pump blood out of the heart. D. The strength of the myocardial cells to shorten with each beat.

The answer is B. Cardiac preload is the amount the ventricles stretch at the end of diastole (the filling or relaxation phase of the heart).

A patient has a blood pressure of 220/140. The physician prescribes a vasodilator. This medication will? A. Decrease the patient's blood pressure and increase cardiac afterload B. Decrease the patient's blood pressure and decrease cardiac afterload C. Decrease the patient's blood pressure and increase cardiac preload D. Increase the patient's blood pressure but decrease cardiac output.

The answer is B. The patient has a high systemic vascular resistance...as evidence by the patient's blood blood....there is vasoconstriction and this is resulting in the high blood pressure. Therefore, right now, the cardiac afterload is high because the ventricle must overcome this high pressure in order to pump blood out of the heart. If a vasodilator is given, it will decrease the blood pressure (hence the systemic vascular resistance) and this will decrease the cardiac afterload. The amount of the pressure the ventricle must pump against will decrease (cardiac afterload decrease) because the blood pressure will go down (hence the systemic vascular resistance).

A patient with cardiogenic shock has a blood pressure of 70/38. In addition, the patient is experiencing dyspnea with a respiratory rate of 32 breaths per minute and has an oxygen saturation of 82% on room air. On auscultation, you note crackles throughout the lung fields. You notify the physician. What order below would you ask for an order clarification? A. Dopamine IV stat B. Normal saline IV bolus stat C. Furosemide IV stat D. Place patient on CPAP (continuous positive airway pressure)

The answer is B. This patient with cardiogenic shock is experiencing a decrease in cardiac output (hence the blood pressure), so an order for Dopamine can help provide a positive inotropic effect (increase the contractility of the heart which will increase stroke volume and cardiac output). The patient is also experiencing pulmonary congestion due to the cardiogenic shock. The heart is failing to pump blood forward, so it is backing up in the lungs. This is leading to an increased respiratory rate, dyspnea, and low oxygen saturation. The order for Furosemide (which is a diuretic) will help remove the extra fluid volume from the lungs and the CPAP (continuous positive airway pressure) will help with oxygenation. The nurse would question the order for a normal saline IV bolus. This bolus would add more fluid to the lungs and further congest the fluids.

The physician orders a urine culture on your patient in room 5505 with a urinary tract infection. In addition, the patient is ordered to start IV Bactrim (Sulfamethoxazole/Trimethoprim). How will you proceed with following this order? A. First, hang the antibiotic, and then collect the urine culture. B. First, hang the antibiotic and when the antibiotic is finished infusing collect the urine culture. C. First, collect the urine culture, and then hang the antibiotic. D. First, collect the urine culture and then hold the dose of the antibiotic until the urine culture is back from the lab.

The answer is C. It is very important to collect the urine culture FIRST and then immediately hang the antibiotic. If the antibiotic is hung first it will decrease the lab's ability to properly identify the bacteria growing in the urine (hence the antibiotic is fighting the infection). It takes approximately 2 days for a urine culture result to come back. Therefore, antibiotic therapy should not be held. The patient needs treatment to prevent the infection from spreading.

A patient with heart failure is taking Losartan and Spironolactone. The patient is having EKG changes that presents with tall peaked T-waves and flat p-waves. Which of the following lab results confirms these findings? A. Na+ 135 B. BNP 560 C. K+ 8.0 D. K+ 1.5

The answer is C. Losartan and Spironolactone can both cause an increased potassium level (hyperkalemia). Losartan is an ARB and Spironolactone is a potassium-sparing diuretic. Therefore, the EKG changes are a sign of a high potassium level (normal potassium level is 3.5-5.1).

A patient is taking Digoxin. Prior to administration you check the patient's apical pulse and find it to be 61 bpm. Morning lab values are the following: K+ 3.3 and Digoxin level of 5 ng/mL. Which of the following is the correct nursing action? A. Hold this dose and administer the second dose at 1800. B. Administer the dose as ordered. C. Hold the dose and notify the physician of the digoxin level. D. Hold this dose until the patient's potassium level is normal.

The answer is C. The patient is Digoxin toxic. A normal Digoxin level is <2 ng/mL. Therefore, the nurse should not administer the dose but hold it and notify the doctor for further orders.

A patient taking Digoxin is experiencing severe bradycardia, nausea, and vomiting. A lab draw shows that their Digoxin level is 4 ng/mL. What medication do you anticipate the physician to order for this patient? A. Narcan B. Aminophylline C. Digibind D. No medication because this is a normal Digoxin level.

The answer is C. The patient is experiencing Digoxin toxicity...therefore the physician will order the antidote for Digoxin which is Digibind.

A 36 year old female, who is 29 weeks pregnant, reports she is experiencing burning when voiding. The physician orders a urinalysis. Which statement by the patient demonstrates she understands how to collect the specimen? A. "I'll hold the cup firmly against the urethra while collecting the sample." B. "I will cleanse back to front with the antiseptic wipe before peeing in the cup." C. "First, I will pee a small amount of urine in the toilet and then collect the rest in the cup." D. "I will be sure to drink a lot of fluids to keep the urine diluted before peeing into the cup."

The answer is C. When collecting a urinalysis it is important to avoid contaminating the sample. So, the patient will collect the urine during mid-stream. The patient will void a small amount in the toilet and then void the rest into the cup (until it is halfway full). The cup should be placed a few inches away from the urethra and prior to voiding the patient should use an antiseptic wipe to cleanse the labia from front to back. It is best to collect the sample when the bladder has been full for 2-3 hours, therefore the urine in concentrated not diluted.

Which patient below is at MOST risk for developing cardiogenic shock? A. A 52-year-old male who is experiencing a severe allergic reaction from shellfish. B. A 25-year-old female who has experienced an upper thoracic spinal cord injury. C. A 72-year-old male who is post-op from a liver transplant. D. A 49-year-old female who is experiencing an acute myocardial infarction.

The answer is D. An acute MI (heart attack) is the main cause of cardiogenic shock. It happens because a coronary artery has become blocked. Coronary arteries supply the heart muscle's cells with oxygenated blood. If they don't receive this oxygenated blood they will die, which causes the heart muscle to quit working (hence pumping efficiently). When the heart muscle fails to pump efficiently, cardiac output fails and cardiogenic shock occur.

A patient is diagnosed with left-sided systolic dysfunction heart failure. Which of the following are expected findings with this condition? A. Echocardiogram shows an ejection fraction of 38%. B. Heart catheterization shows an ejection fraction of 65%. C. Patient has frequent episodes of nocturnal paroxysmal dyspnea. D. Options A and C are both expected findings with left-sided systolic dysfunction heart failure.

The answer is D. Both Options A and C are correct. Option B is a finding expected in left-sided DIASTOLIC dysfunction heart failure because the issue is with the ability of the ventricle to FILL properly...therefore a patient usually has a normal ejection fraction. Remember a normal EF is >60% in a healthy heart.

Select the statement below that best describes cardiac afterload: A. It's the volume amount that fills the ventricles at the end of diastole. B. It's the volume the ventricles must work against to pump blood out of the body. C. It's the amount of blood the left ventricle pumps per beat. D. It's the pressure the ventricles must work against to open the semilunar valves so blood can be pumped out of the heart.

The answer is D. Cardiac afterload is the pressure the ventricles must work against to pump blood out of the heart by opening up through the semilunar valves. So, it's the pressure the ventricles must overcome to open the semilunar valves to push blood out of the heart.

Which of the following tests/procedures are NOT used to diagnose heart failure? A. Echocardiogram B. Brain natriuretic peptide blood test C. Nuclear stress test D. Holter monitoring

The answer is D. Options A, B, and C are all used to diagnose heart failure...however a holter monitor is not. A holter monitor is used to monitor a patient's heart rate and rhythm.

Which of the following is a late sign of heart failure? A. Shortness of breath B. Orthopnea C. Edema D. Frothy-blood tinged sputum

The answer is D. Shortness of breath, orthopnea, and edema are EARLY signs and symptoms. Frothy-blood tinged sputum is a late sign.

These drugs are used as first-line treatment of heart failure. They work by allowing more blood to flow to the heart which decreases the work load of the heart and allows the kidneys to secrete sodium. However, some patients can develop a nagging cough with these types of drugs. This description describes? A. Beta-blockers B. Vasodilators C. Angiotensin II receptor blockers D. Angiotensin-converting-enzyme inhibitors

The answer is D. This is a description of ACE inhibitors

You're assessing your patients during morning rounding. Which patient below is at MOST risk for developing a urinary tract infection? A. A 25 year old patient who finished a regime of antibiotics for strep throat 10 weeks ago. B. A 55 year old female who is post-opt day 7 from hip surgery. C. A 68 year old male who is experiencing nausea and vomiting. D. A 87 year old female with Alzheimer's disease who is experiencing bowel incontinence.

The answer is D. This patient has many risks factors for developing a UTI. The patient is postmenopausal which leads to flora changes in the vaginal area. This can increase the risk of developing a UTI. In addition, bowel incontinence increases the risk of a UTI due to the anatomy of the female (short urethra) and the close proximity between the rectum to the urethra. Also, patients with Alzheimer's disease may experience bladder retention due to the inability to communicate the need to void which increases the amount of time the urine is left in the bladder. Option A is wrong because although antibiotics can increase the risk of developing a UTI the patient finished the antibiotics 10 weeks ago. Options B and C are wrong because they do not provide enough information to determine if the patients are at risk for a UTI.

What type of heart failure does this statement describe? The ventricle is unable to properly fill with blood because it is too stiff. Therefore, blood backs up into the lungs causing the patient to experience shortness of breath. A. Left ventricular systolic dysfunction B. Left ventricular ride-sided dysfunction C. Right ventricular diastolic dysfunction D. Left ventricular diastolic dysfunction

The answer is D. This statement describes left ventricular DIASTOLIC dysfunction.

True or False: If a patient with cardiogenic shock is given a medication that will decrease cardiac afterload, it will decrease stroke volume.

The answer is FALSE. If a patient with cardiogenic shock is given a medication that will decrease cardiac afterload, it will INCREASE (not decrease) stroke volume. HOW? Remember afterload is the resistance the ventricle must pump against to get blood out of the heart. If we DECREASE the resistance the heart must pump against (make it easier for the heart squeeze blood out of the ventricle), it will increase how much blood it will pump out, hence increase the stroke volume.

True or False: Patients with left-sided diastolic dysfunction heart failure usually have a normal ejection fraction.

The answer is TRUE. Patient with left-sided DIASTOLIC dysfunction heart failure normally have a normal ejection fraction. However, patients with left-sided SYSTOLIC dysfunction heart failure usually do not because the heart is unable to CONTRACT efficiently rather than fill properly as with diastolic dysfunction.

A 74 year old female presents to the ER with complaints of dyspnea, persistent cough, and unable to sleep at night due to difficulty breathing. On assessment, you note crackles throughout the lung fields, respiratory rate of 25, and an oxygen saturation of 90% on room air. Which of the following lab results confirm your suspicions of heart failure? A. K+ 5.6 B. BNP 820 C. BUN 9 D. Troponin <0.02

The answer is option B. BNP (b-type natriuretic peptide) is a biomarker released by the ventricles when there is excessive pressure in the heart due to heart failure. <100 no failure, 100-300 present, >300 pg/mL mild, >600 pg/mL >moderate, 900 pg/mL severe

On your nursing care plan for a patient with a urinary tract infection, which of the following would be appropriate nursing interventions? SELECT-ALL-THAT-APPLY: A. Encourage voiding every 2-3 hours while awake. B. Restrict fluid intake to 1-2 liters per day. C. Monitor intake and output daily. D. The patient verbalizes the importance of using vaginal sprays to decrease reoccurrence of urinary tract infections prior to discharge home.

The answers are A and C. Option D is wrong become this is not a nursing intervention but a patient goal. In addition, it is an incorrect patient goal for preventing UTIs. Vaginal sprays should be avoided. Option B is wrong because fluid intake should be encouraged of 2-3 liters per day. This will help the urinary system flush out the presenting infection.

What conditions below can result in an increased cardiac afterload? Select all that apply: A. Vasoconstriction B. Aortic stenosis C. Vasodilation D. Dehydration E. Pulmonary Hypertension

The answers are A, B, and E. Vasoconstriction increases systemic vascular resistance which will increase cardiac afterload. It will increase the pressure the ventricle must pump against to open the semilunar valves to get blood out of the heart. Aortic stenosis creates an outflow of blood obstruction for the ventricle (specifically the left ventricle) and this will increase the pressure the ventricle must pump against to get blood out through the aortic valve. Pulmonary hypertension increases pulmonary vascular resistance which will increase the pressure the right ventricle must overcome to open the pulmonic valve to get blood out of the heart....all of this increase cardiac afterload.

Select all the correct statements about educating the patient with heart failure: A. It is important patients with heart failure notify their physician if they gain more than 6 pounds in a day or 10 pounds in a week. B. Patients with heart failure should receive an annual influenza vaccine and be up-to-date with the pneumonia vaccine. C. Heart failure patients should limit sodium intake to 2-3 grams per day. D. Heart failure is exacerbated by illness, too much fluid or sodium intake, and arrhythmias. E. Patients with heart failure should limit exercise because of the risks.

The answers are B, C, and D. Option A is wrong because heart failure patients should notify their doctor if they gain 2-3 pounds in a day or 5 pounds in a week, and option E is wrong because exercise is important for heart failure patients to help strengthen the heart muscle...so they should exercise as tolerated.

Stroke volume plays an important part in cardiac output. Select all the factors below that influence stroke volume: A. Heart rate B. Preload C. Contractility D. Afterload E. Blood pressure

The answers are B, C, and D. Preload, afterload, and contractility all have a role with influencing stroke volume.

You're assessing your patient with cardiogenic shock, what signs and symptoms do you expect to find in this condition? Select all that apply: A. Warm, flushed skin B. Prolonged capillary refill C. Urinary output >30 mL/hr D. Systolic blood pressure <90 mmHg E. Crackles in lung fields F. Dyspnea D. Decreased BUN and creatinine G. Strong peripheral pulses H. Chest pain

The answers are B, D, E, F, and H. Signs and symptoms of cardiogenic shock will be related to LOW cardiac output and decreased perfusion to organs/tissues. Capillary refill will be prolonged >2 seconds, urinary output will be <30 mL/hr, systolic blood pressure will be <90 mmHg, pulmonary edema will present with fluid in the lungs (hence crackles in the lungs), dyspnea, and chest pain (due to decreased blood flow to the heart muscle).

Cardiac output is equal to the heart rate multiplied by the stroke volume. Treatment for cardiogenic shock includes medications that increase cardiac output. One of the factors that help determine cardiac output is stroke volume. Select all the factors that determine stroke volume? A. Cardiac Index B. Preload C. Pulmonary capillary wedge pressure D. Afterload E. Heart rate F. Contractility

The answers are B, D, and F. Cardiac output is determined by the person's heart rate times the stroke volume. Stroke volume is the amount of blood pumped from the left ventricle with each BEAT (50-100 ml). It's determined by the preload, afterload, and contractility of the heart. These factors in a patient with cardiogenic shock can be manipulated with medications to increase the cardiac output.

Which of the following are NOT typical signs and symptoms of right-sided heart failure? Select-all-that-apply: A. Jugular venous distention B. Persistent cough C. Weight gain D. Crackles E. Nocturia F. Orthopnea

The answers are B, D, and F. Persistent cough, crackles (also called rales), and orthopnea are signs and symptoms of LEFT-sided heart failure...not right-sided heart failure.

Which treatments below would decrease cardiac preload? Select all that apply: A. IV fluid bolus B. Norepinephrine C. Nitroglycerin D. Furosemide

The answers are C and D. Nitroglycerin is a vasodilator that will dilate vessels, which will decrease venous return to the heart and this will decrease preload. Furosemide is a diuretic which will remove extra fluid from the body via the kidneys. This will decrease venous return to the heart and decrease preload. An IV fluid bolus and Norepinephrine (a vasoconstrictor) will increase venous return to the heart and increase preload.

Which medications below are used in cardiogenic shock that provide a positive inotropic effect on the heart? Select all that apply: A. Nitroglycerin B. Sodium Nitroprussidde C. Dobutamine D. Norepinephrine E. Dopamine

The answers are C and E. Dobutamine and Dopamine are vasopressors that have a POSITIVE inotropic effect on the heart. This means these medications increase the strength of the heart's contractions (increases contractility), which increases stroke volume.

A patient is being treated for cardiogenic shock. Which statement below best describes this condition? Select all that apply: A. "The patient will experience an increase in cardiac output due to an increase in preload and afterload." B. "A patient with this condition will experience decreased cardiac output and decreased tissue perfusion." C. "This condition occurs because the heart has an inadequate blood volume to pump." D. "Cardiogenic shock leads to pulmonary edema."

The answers are: B and D. Cardiogenic shock occurs when the heart can NOT pump enough blood to meet the perfusion needs of the body. The cardiac output will be DECREASED, which will DECREASE tissue perfusion and cause cell injury to organs/tissues. In this condition, the heart is WEAK and can't pump blood out of the heart. This can be due to either a systolic (contraction) or diastolic (filling) issue along with a structural or dysrhythmia issue. In cardiogenic shock, there is NOT an issue with blood volume, but there is a problem with the heart itself.

Which of the following patients are MOST at risk for developing heart failure? Select-all-that-apply: A. A 69 year old male with a history of alcohol abuse and is recovering from a myocardial infarction. B. A 55 year old female with a health history of asthma and hypoparathyroidism. C. A 30 year old male with a history of endocarditis and has severe mitral stenosis. D. A 45 year old female with lung cancer stage 2. E. A 58 year old female with uncontrolled hypertension and is being treated for influenza.

The answers to this question are options: A, C, E. These patients are at most risk for heart failure. Remember risks factor for developing heart failure include: remember the mnemonic FAILURE: Faulty heart valves ( Option C mitral stenosis in this case), Arrhythmias, Infarction (Option A), Lineage, Uncontrolled hypertension (Option E), Recreational drug usage, Evaders (Option E with influenza)

A primary care nurse auscultates a patient with chronic heart failure and hears an audible S4 before the S1. The nurse expects this finding to correlate with which type of heart failure? a. Left-sided heart failure b. Right-sided heart failure c. Heart failure with reduced ejection fraction d. Heart failure with preserved ejection fraction

Trd. · An S4 is most associated with diastolic heart failure which has a preserved ejection fraction (the problem is filling, not ejection of blood). · An S3 is most associated with systolic heart failure which has a reduced ejection fraction (because the blood can't get out of the ventricle) · The left and right sided heart failures predict where blood will backflow but have no bearing on whether ejection fraction is reduced or preserved Trick: di-a-stol-ic has 4 syllables= s4 heart sound sys-tol-ic has 3 syllables= s3 heart sound

True or False: The left anterior descending coronary artery provides blood supply to the left ventricle, front of the septum and part of the right ventricle.

True...the LAD (left anterior descending artery) provides blood supply to the left ventricle, front of the septum and collateral circulation to the right ventricle.

In order to counteract the body's homeostatic response to decreased cardiac output and to prevent ventricular remodeling, patients with heart failure may be prescribed which class of medications? a. Angiotensin converting enzyme inhibitors b. Beta-blockers c. Calcium channel blockers d. Diuretics

a. Angiotensin converting enzyme inhibitors

Which of the following systems of the body are affected by hypertension? a. Cardiovascular, brain, kidney, eyes b. Cardiovascular, gastrointestinal, reproductive, and kidney c. Brain, respiratory, kidney, cardiovascular d. None of the options are correct

a. Cardiovascular, brain, kidney, eyes (fifth 'organ' Peripheral vasculature)

The following are associated with Right-sided HF: SATA a. Frothy Sputum production b. JVD c. Increased peripheral edema d. pulmonary HTN e. pulmonary edema

b, c, d In right sided heart failure, fluid backs up into the peripheral vasculature, resulting in fatigue, venous jugular distention, peripheral edema, ascites, hepato- and spleno-megaly, anorexia, Gi distress, weight gain, dependednt edema, and increased pulmonary venous pressure. Pulmonary HTN is also associated with right-sided HF (it may be a precipitating factor to R sided HF).

Which of the following patients does not have a risk factor for hypertension? a. A 25 year old male with a BMI of 35. b. A 35 year old female with a total cholesterol level of 100. c. A 68 year old male who reports smoking 2 packs of cigarettes a day. d. A 40 year old female with a family history of hypertension and diabetes.

b. A 35 year old female with a total cholesterol level of 100. (this is normal)

A 78-yr-old man with a history of diabetes has confusion and temperature of 104°F (40°C). There is a wound on his right heel with purulent drainage. After an infusion of 3 L of normal saline solution, his assessment findings are BP 84/40 mm Hg; heart rate 110; respiratory rate 42 and shallow; CO 8 L/min; and PAWP 4 mm Hg. This patient's symptoms are most likely indicative of a. sepsis. b. septic shock. c. multiple organ dysfunction syndrome. d. systemic inflammatory response syndrome.

b. septic shock. Temperature, hypotensive despite fluid resuscitation, high hr and high RR (SNS stimulation), high CO (early sign of septic shock, later it will drop)

The nurse teaches the female patient who has frequent UTIs that she should a. take tub baths with bubble bath. b. void before and after sexual intercourse. c. take prophylactic sulfonamides for the rest of her life. d. restrict fluid intake to prevent the need for frequent voiding.

b. void before and after sexual intercourse.

The nurse is assessing a female patient at the neighborhood clinic. The patient reports "feeling tired all the time." The nurse knows that fatigue may be an underlying symptom of which condition? a. Pneumonia b. Peptic ulcer disease c. Myocardial infarction d. Ischemia

c Fatigue is an atypical symptom of myocardial infarction in women. Ischemia is associated with pain. Pneumonia is associated with pain and shortness of breath. Peptic ulcer disease is associated with pain and intestinal discomfort.

Name a central acting α2-Adrenergic Agonist that reduces peripheral sympathetic tone and produces vasodilation, and is used for hypertension: c________

clonidine

A 53 year old caucasian male with DM type 2 and CKD is diagnosed with primary hypertension. The nurse anticipates teaching the patient about which first-line medication? a. Sodium nitroprusside b. Hydrochlorothiazide c. Metoprolol d. Lisinopril

d. ACE Inhibitors and ARBs are first line of treatment for patients of any race with DM and CKD. Caucasian patients without CKD/DM would be treated with thiazides as first line treatments, as well as ACE inhibitors, ARBs, and CCBs. Black patients without renal issues/DM would avoid ACE inhibitors and ARBs and be treated with thiazide and/or CCBs

A nurse is assessing a client with a diagnosis of early left ventricular heart failure. Specific to this type of heart failure, what statement by the client would the nurse expect? · "When I eat a large meal, I feel bloated." · "I am tired at the end of the day." · "Both of my ankles are swollen." · "I have trouble breathing when I walk rapidly."

· "I have trouble breathing when I walk rapidly." Dyspnea on exertion often occurs with left ventricular heart failure because the heart is unable to pump enough oxygenated blood to meet the energy requirements for muscle contractions related to the activity. The statement "My ankles are swollen" is more likely with right ventricular heart failure. The statement "I am tired at the end of the day" is not specific to left ventricular heart failure. The statement "When I eat a large meal, I feel bloated" is not specific to left ventricular heart failure.

A client is taking furosemide and digoxin for heart failure. Why does the nurse advise the client to drink a glass of orange juice every day? · "To maintain a healthy potassium level." · "To prevent increased sodium levels." · To limit the drugs' synergistic effects." · "To correct for dehydration."

· "To maintain a healthy potassium level." Orange juice is an excellent source of potassium. Furosemide promotes excretion of potassium, which can result in hypokalemia. Digoxin toxicity can occur in the presence of hypokalemia. Neither drug increases sodium levels. Digoxin does not potentiate the action of furosemide; therefore, the client should not experience dehydration. Orange juice will not prevent an interaction between digoxin and furosemide.

The nurse is assessing a client with the diagnosis of chronic heart failure. Which clinical finding should the nurse expect the client to experience? · Palpitations in the chest when resting · Dependent edema in the evening · Chest pain that decreases with rest · Frequent coughing with yellow sputum

· Dependent edema in the evening Decreased cardiac output causes fluid retention, which results in dependent edema; this is often noticed in the evening after the client has been standing or sitting for prolonged periods. Chest pain is indicative of cardiac ischemia. Palpitations are indicative of cardiac dysrhythmias. Coughing with yellow sputum is indicative of an infectious process in the respiratory tract; pink, frothy sputum is associated with pulmonary edema that can result from heart failure.

A patient is experiencing a myocardial infarction. What should the nurse identify as the primary cause of pain experienced by a patient with a coronary occlusion? · Irritation of nerve endings in the cardiac plexus · Blocking of the coronary veins · Heart muscle ischemia · Arterial spasm

· Heart muscle ischemia

A patient with a history of coronary artery disease is prescribed atenolol. What should the nurse monitor to determine the therapeutic effect of the medication? · Heart rate · Pulse oximetry · Temperature · Respirations

· Heart rate

A nurse is teaching a group of patients about risk factors for heart disease. Which factor will the nurse include that increases a patient's risk for myocardial infarction? · Family history of breast cancer · Hypertension · Asian-American ancestry · Increased high density lipoproteins (HDL)

· Hypertension

Which urinalysis finding indicates a urinary tract infection? · Presence of ketones · Presence of bilirubin · Presence of crystals · Presence of leukoesterase

· Presence of leukoesterase Leukoesterases are released by white blood cells as a response to an infection or inflammation. Therefore, the presence of this chemical in urine indicates a urinary tract infection. The presence of crystals in the urine indicates that the specimen had been allowed to stand. Presence of bilirubin in the urine indicates anorexia nervosa, diabetic ketoacidosis, and prolonged fasting. The presence of ketones indicates diabetic ketoacidosis.

The nurse is caring for a client who had a massive myocardial infarction and developed cardiogenic shock. Which clinical manifestation supports this diagnosis? · Warm, flushed skin · Increased blood pressure · Deep respirations · Rapid pulse

· Rapid pulse The heart rate increases (tachycardia) in an attempt to meet the body's oxygen demands and circulate blood to vital organs; the pulse is weak and thready because of peripheral vasoconstriction. The respirations are rapid and shallow, not deep. The skin is cold and clammy because of vasoconstriction caused by the shunting of blood to vital organs. The blood pressure is decreased, not increased, because of continued hypoperfusion and multiorgan failure.


Conjuntos de estudio relacionados

Chapter 11: Self Identity and Personality

View Set

ARM 54 Assignment 4 Practice exam

View Set

E-Marketing Final Exam- Chapter 12-E-Marketing Communication: Owned Media

View Set

history study guide (kahoot + tci lesson game)

View Set

LSAT Test Masters Question Types

View Set

Respiration: Rate, Rhythm, and Effort Quiz

View Set

Life Insurance Exam: Chapter 5 | Annuities

View Set

CCNA 2 EXAM 1-4 STUDY Guide copy 2

View Set